Söker efter Einstein och relativitet

 

114 frågor/svar hittade

Universum-Solen-Planeterna [96]

Fråga:
Vad är ett svart hål?
/

Svar:
För att kunna förstå vad ett svart hål är så måste man kunna något om den allmänna relativitetsteorin. Denna teori behandlar gravitationen. Enligt Newton så påverkar t ex solen jorden med en kraft som gör att dess bana kröks. I Einsteins teori däremot så blir själva rummet krökt på grund av gravitationen och jorden rör sig "lokalt rakt fram" på denna krökta yta. Tänk på en myra som kryper, På en plan yta kryper den hela tiden rakt fram. Kryper den nära skaftet på ett äpple så kan den krypa i en cirkel trots att den hela tiden är dum nog att tro att den kryper rakt fram.

Alltså: Stora massor gör att rummet kröks.

Ett svart hål är ett ställe där rummets krökning är oändligt stort. Detta kan bara ske i en punkt. Man kan enklast säga att i denna punkt existerar inte tid och rum längre. I mitten av ett svart hål finns en singularitet, dvs en punkt i rum-tiden där dess krökningsradie är oändligt stor. Allt som kommer i närheten av denna punkt måste röra sig in i och "uppslukas" av punkten. Det svarta hålet omger sig av en tänkt, sfärisk yta, "händelsehorisonten" som är sådan att allt som kommer innanför denna yta måste försvinna in i hålet.

Man kan jämföra det svarta hålet med en mycket effektiv dammsugare som suger i sig allt som kommer i närheten.

Ett svart hål uppstår då en tillräckligt stor stjärna kollapsar under sin egen tyngd. Det finns också teorier om ursprungliga svarta hål som skulle ha skapats redan vid Big-Bang. Det är mycket osäkert om det finns sådana.

Se även Black Holes - Portals into the Unknown .

Läs: I den trevliga boken "Tid utan ände" av Coveney och Highfield finns en beskrivning av svarta hål.

Nyckelord: relativitetsteorin, allmänna [33]; svart hål [51];

*

Universum-Solen-Planeterna [98]

Fråga:
Finns det vita hål? Hur uppkomer dom i så fall?
Varför heter det svarta hål? Är hålet svart?
Vill ha lite mer information om Einsteins allmänna relativitetsteori.
/

Svar:
Vita hål är en teoretisk modell av ett tidsspeglat svart hål. I många fall t ex för enkel kaströrelse så kan den tidsomkastade rörelsen också ske. (Filma en boll som flyger med låg hastighet. Du kan köra filmen både fram och baklänges utan att få ett skeende som strider mot fysikens lagar.) Det finns inga bevis för att det skulle finnas vita hål. Ett vitt hål skulle enligt vissa teorier uppkomma samtidigt som ett svart hål. Den materia som sugs in i det svarta hålet skulle kastas ut någon annanstans i universum, eller i ett annat universum. Det senare är mycket hypotetiskt då vi inte vet om det finns flera universa!

Termen svart hål kommer av att inget ljus kan lämna området innanför händelshorisonten. Hålet är verkligen svart, men utanför hålet kan gaserna som sugs in mot hålet lysa kraftigt! Det heter vitt hål eftersom det är motsatsen till svart hål.

Läs: Du kan läsa om allmän relativitetsteori i den tidigare nämnda boken "Tid utan ände" av Coveney & Highfield eller i "Kvarken och Universum" av Bengtsson, Gustafson och Gustafson.

*

Kraft-Rörelse [157]

Fråga:
Vad är det för skillnad på gravitation och acceleration i fritt fall?
/

Svar:
Ett föremål vid jordytan påverkas av tyngdkraften som har storleken m·g där m är föremålets massa. Tyngdfaktorn g har enheten N/kg och det gäller att g = 9,82 N/kg. (Tyngdfaktorn ökar när man rör sig norrut och det kanske är så att g = 9,83 N/kg hos er i Vilhelmina, jag vet inte riktigt!) Om man släpper ett föremål så att det faller fritt så får det en konstant acceleration a = 9,82 m/s/s precis lika stor som tyngdfaktorn. Kollar man enheterna så finner man enheterna för både tyngdfaktorn och accelerationen vid fritt fall är samma. Alltså gäller att a = g. Detta framgår också av kraftlagen: F = m·a. Sätt F = m·g så får Du att a = g. För Einstein betydde denna likhet mycket och han tog den som en av utgångspunkterna för sin allmänna relativitetsteori.

*

Ljud-Ljus-Vågor [175]

Fråga:
Hur kommer det sig att fotoner kan påverkas av gravitation?
/

Svar:
Man kan se det på två sätt:

För det första så har fotonen energi. Eftersom energi och massa i grunden är samma sak så har fotonen även massa och påverkas därmed av gravitationen.

Det andra synsättet är det som Einstein införde i och med att han konstruerade den allmänna relativitetsteorin. I denna teori är rummet krökt på grund av gravitationen och partiklar som inte påverkas av några andra krafter "går rakt fram" i detta rum.

*

Blandat [396]

Fråga:
Hej jag har läst om lite grejor på eran sida och det har varit kul. Jag skulle vilja veta om ni vet av någon bra internetresurs som tar upp Albert Einstein och förklarar saker kring hans relativitetsteori. Om inte så kan ni väl berätta för mig. Trots allt så skrev väl Einstein om den för "lekmän"!
/

Svar:
Center for History of Physics är en bra startpunkt.

Läs: Det finns många populärvetenskapliga böcker om relativitetsteori. En är Nigel Calder: "Einsteins Universum"

*

Blandat [404]

Fråga:
Varifrån kommer egentligen konstanten i Newtons gravitationslag?
Och för all del andra s k naturkonstanter också?
Beror deras värde av hur vi valt grundenheter: meter sekund o s v.
/

Svar:
Fysikens mest grundläggande samband kan man varken härleda eller säga att de "kommer någonstans ifrån". Konstanten i Newtons gravitationslag anger hur stark gravitationskraften är och på fysikens nuvarande nivå kan vi inte relatera den till något mer grundläggande. Visserligen finns Einsteins allmänna relativitetsteori men även i denna teori finns det en konstant vars värde endast kan bestämmas genom mätningar.

Värdet på alla konstanter som har en dimension beror av valet av grundenheterna.

*

Blandat [405]

Fråga:
Jag skulle vilja ha tips på några berömda fysiker (gärna svenska) och deras upptäckter, och om det finns något material om detta på internet.
/   

Svar:
Den bästa källan för information är uppslagsverk, speciellt den nya Nationalencyklopedin . Slå upp nedanstående namn i den! Se även boken "Om mått och män" av Sten von Friesen och Kosmos 1987.

Vi börjar med några svenska fysiker:

Tycho Brahe (Han var dansk men levde i nuvarande Skåne) Upptäckte supernovaexplosioner, gjorde noggranna och systematiska mätningar av stjärnornas och planeternas positioner på himlavalvet.

Anders Celsius Införde temperaturskala.

Manne Siegbahn Undersökning av röntgenstrålning

Janne Rydberg Atomspektra

Några utländska:

Isaac Newton Grundlade den klassiska mekaniken och skapade gravitationsteorin.

Michael Faraday Utforskade elektricitetsläran.

James Clark Maxwell Teorin för elektromagnetismen.

Niels Bohr Atomteori och förklarade det periodiska systemet.

Albert Einstein Skapade relativitetsteorin

Några kvinnor:

Marie Curie Undersökte radioaktivitet

Lise Meitner Upptäckte fission.

*

Universum-Solen-Planeterna [473]

Fråga:
Har man någonsin lyckats registrera en "gravitationsvåg"? Jag tycker att med denna massdilationsutbredning borde det även medfölja en tidsdilationsutbredning såväl som en längdkontraktionutbredning vilket enligt min åsikt skulle göra det omöjligt att registrera en sådan händelse. Allt enligt gängse formler inom relativitetsteorin.
/

Svar:
Nobelpris i fysik 1993
Gravitationsvågor
Russell A. Hulse, Joseph H. Taylor, Jr
"for the discovery of a new type of pulsar, a discovery that has opened up new possibilities for the study of gravitation"

Man har ännu inte lyckats detektera någon gravitationsvåg och det kommer förmodligen att dröja ännu ett tag innan man har konstruerat en tillräckligt känslig detektor. Gravitationsvågor utsänds t ex av två stjärnor som cirkulerar runt varandra. Man har observerat att sådana system förlorar energi vilket tolkas som ett indirekt bevis för att dessa vågor existerar. Nobelpriset i fysik delades 1993 ut till forskare som studerat detta för neutronstjärnor, se nedanstående bild och The Nobel Prize in Physics - Laureates och The Binary Pulsar PSR 1913+16 .

Se vidare PSR_1913+16 och Gravitational_wave .

Figuren nedan visar hur omloppstiden hos systemet PSR 1913+16 minskar (punkter med felstaplar). Den heldragna linjen är förutsägelsen från den allmänna relativitetsteorin. Överensstämmelsen och avsaknaden av en alternativ modell som kan förklara vart den förlorade energin tar vägen är alltså ett mycket starkt indirekt bevis för att gravitationsvågor existerar.

Läs: I Svenska fysikersamfundets årsbok "Kosmos" från år 1994 finns en beskrivning av den forskning som ledde till de med Nobelpriset belönade upptäckterna. Speciellt starka gravitationsvågor kommer från supernovaexplosioner eller då två svarta hål kolliderar. Även när de stora svarta hål, som troligtvis finns i centrum av de flesta galaxer, "glufsar i sig" tunga stjärnor så får man gravitationsvågor med stor amplitud. Vad är en gravitationsvåg? Enligt Einstein är den fyrdimensionella rum-tiden krökt. För närmare diskussion av detta se tidigare svar! En gravitationsvåg är en störning i krökningen som utbreder sig. Där vi befinner oss så är rum-tiden nästan helt plan. När en gravitationsvåg kommer så förändras krökningen momentant, ungefär som krökningen på vattenytan ändras när en våg passerar. Detta leder till att kroppar drar ihop sig eller sträcker ut sig. I en detektor för sådana vågor använder man denna effekt. Om jag förstår Din fråga rätt så menar Du att detta inte går att detektera. För att mäta en längdförändring hos detektorn måste man ju i princip använda en måttstock. Men måttstocken påverkas ju på samma sätt som detektorn. Alltså går det inte att detektera någon våg. Det är ett helt relevant påpekande. I de detektorer som byggs mäter man längdförändringar med hjälp av interferens mellan ljusstrålar. Om man räknar noga på hur ljuset påverkas av gravitationsvågen så visar det sig att man verkligen får en effekt. Den är dock oerhört liten och mycket svår att detektera.

Läs: Jag kan rekommendera boken "Black holes and time warps" av Kip S. Thorne. Det är den bästa populärvetenskapliga bok jag (GO) läst och den innehåller bl a ett kapitel om gravitationsvågor och detektionen av dem.

Se fråga 20117 för direkt detektion av gravitationsvågor.



/Gunnar O/Peter E

Nyckelord: gravitationsvågor [19]; relativitetsteorin, allmänna [33]; neutronstjärna [11];

1 http://www.forskning.se/nyheterfakta/nyheter/pressmeddelanden/tungpulsargereinsteinrattansalange.html
2 http://www.sciencemag.org/content/340/6131/1233232

*

Kraft-Rörelse [3652]

Fråga:
Hej! Skulle ni kunna förklara, så gott det nu går, vad S.Hawking menar med "hoprullade dimensioner" (i supersträng-sammanhang)? Sen undrar jag om det finns/fanns någon människa som kunnat föreställa sig fler än tre dimensioner, och verkligen "sett" det framför sig. Är det möjligt? Kunde tex Einstein det?
/Nina a, Österport, Ystad

Svar:
Vi upplever normalt 3 rumsdimensioner och tiden. Matematiskt är det inga problem att behandla hur många dimensioner som helst. Supersträngteori (se The Official String Theory Web Site ) försöker förena den allmänna relativitetsteorin med kvantmekaniken och föra in alla fysikaliska fenomen under en gemensam teori. Där behövs flera rumsdimensioner. Alla utom 3 skulle vara fördolda för oss för att rummets krökning i de övriga skulle vara enorm. 
/KS

*

Kraft-Rörelse [9958]

Fråga:
Hej. Jag håller för tillfället på att skriva ett specialarbete om Albert Einstein. Uppenbarligen är gubben oerhört respekterad och har ju en rejäl legendstatus, men finns det egentligen något som säger att Einstein var ovanligt intelligent?

Jag har exempelvis sett att man i dagsläget ibland jämför Stephen Hawking med Einstein. Nog måste det väl ha funnits andra fysiker under 1900-talet som gjorde liknande bedrifter som Einsten, eller beror hans status just bara på att han "rättade" Newton?
/Johan E, Staffanskolan, Ljusne

Svar:
Relativitetsprincipen:

Naturlagarna är samma i alla inertialsystem (system i likformig rörelse).

En teori där relativitetsprincipen gäller, kan kallas en relativitetsteori. Newtons mekanik, till exempel, är en relativitetsteori. Under 1800-talet arbetade fysikerna mycket med elektriska och magnetiska fenomen. Det kulminerade i en matematiskt elegant sammanfattning som kallas Maxwells Ekvationer. Det fanns ett stort problem. Denna teori uppfyllde inte relativitetsprincipen.

Man arbetade mycket med att modifiera ekvationerna, men utan framgång. Allt detta detta är numera bortglömt. Det ledde ju bara till återvändsgränder. År 1887 publicerade Michelson och Morley resultatat av ett experiment, som visade att ljushastigheten är densamma i olika inertialsystem. Detta strider mot år varagsuppfattning, vi är vana vid att kunna addera hastigheter. Vad Einstein gjorde, var att koppla ihop detta med problemet med Maxwells Ekvationer. Han han hittade en lösning, som publicerades år 1905 i Zeitschrift für Phüsik. Översatt till svenska är titeln: "Om rörliga kroppars elektrodynamik". Det är vad vi idag kallar den speciella relativitetsteorin.

Man kan inte säga att Einstein var särskilt radikal vad matematiken beträffar. Det har lett till en del problem, tvillingparadoxen till exempel. Hermann Minkowski hittade några år senare en elegant matematisk form, som förövrigt Einstein helt accepterade, när han 1916 införlivade gravitationen. Den teorin brukar kallas den allmänna relativitetsteorin. Det finns än idag inte någon iakttagelse som motsäger dessa teorier. Det är väl ett gott betyg. Däremot kan man inte säga att Einstein gjorde några betydande insatser inom kvantmekaniken. Han var till och med rätt skeptisk mot vissa företeelser där, till exempel slupmässigheten, som vi numera vet är av fundamental betydelse.

Det som står i fetstil lönar det sig att söka på i denna databas. Du kan också gå till Nationalencyklopedin .
/KS

*

Kraft-Rörelse [782]

Fråga:
Om två bilar åker ifrånvarandra med hastigeterna 75 km/h. Då avlägsnar ju dom sig från varandra med 150 km/h Om två "planeter" åker i från varandra med 60% av ljusets hastighet då kan do ju inte avlägsna sig från varandra med 120 % av ljusetshastiget. Hur gör man för att räkna ut det?
/Markus H, Dergården, Lerum

Svar:
Man får använda "Einsteins formel för addition av hastigheter" som bygger på den speciella relativitetsteorin. Om vi kallar hastigheterna v1 och v2, så blir det relativa hastigheten

(v1+v2)/(1+v1v2/c2).

För ditt exempel v1=v2=0.6c blir resultatet 1.2c/(1+0.36)=0.88c. Observera att om en av hastigheterna är c blir den relativa hastigheten

(c+v2)/(1+c*v2/c2) = (c+v2)/(1+v2/c) = c(c+v2)/(c+v2) = c

alltså summan blir aldrig högre än c.
/Peter Ekström

Nyckelord: relativitetsteorin, speciella [45]; addition av hastigheter [2]; ljushastigheten [24];

*

Blandat [1074]

Fråga:
Jag vill veta vad det finns för några bevis för att relativitetsteorin stämmer. Jag vill också veta var jag kan läsa ingående om dessa bevis.
/Gustav A, Alströmergymnasiet, Alingsås

Svar:
Det finns många både direkta och indirekta bevis för Einsteins relativitetsteori. Den speciella relativitetsteorin behandlar tid och rum samt energi och massa. Där finns det bland annat följande bevis:

  • Tiden går långsammare för partiklar i rörelse. Detta har bevisats i en mängd experiment i partikelacceleratorer och för myoner från kosmisk strålning (se fråga 2697 ).
  • Massans och energins ekvivalens har bevisats i kärnfysiken där man kan mäta både massa och energi för olika atomkärnor.
Den allmänna relativitetsteorin handlar om gravitationen och hur den påverkar rummets krökning. Bevisen kommer här från astrofysiken:
  • Man har upptäckt svarta hål som förutsägs av teorin.
  • Dubbelstjärnesystem förlorar energi i enlighet med teorins förutsägelser, se fråga 473 .

Listan kan göras mycket längre. Läs någon av de populära böcker som finns om relativitetsteorin, till exempel "Einsteins universum" av Calder. Se även Allmän relativitetsteori och kosmologi .
/GO

Nyckelord: relativitetsteorin, allmänna [33]; relativitetsteorin, speciella [45];

*

Kraft-Rörelse [1135]

Fråga:
Hej tack för svaret på min förra fråga! Jag undrar lite om formeln F=k*m1*m2/(r*r) (även F=m*a). Påverkar kraften F båda massorna lika mycket? om man har två massor och räknar ut kraften mellan vardera massa och en tredje massa typ: F=k*m1*m3/(r*r) F=k*m2*m3/(r*r) där den tredje massan är konstant. Man får då fram resultatet att m1 och m2 får samma acceleration mot m3 (enl. F=m*a) men om man vänder på det hela och räknar på hur stor acceleration m3 har mot m1 resp. m2 så får man fram att accelerationen i dessa två fall är olika om m1 och m2 är olika stora. Slutsatsen bör då vara att m3 rör sig inte med samma hastighet mot både m1 och m2. Man har sedan två accrelerationer i vardera fall att räkna på m3 mot m1 och m1 mot m3 resp. m3 mot m2 och m2 mot m3. det är här mitt problem är, om jag adderar accerelerationerna så far jag fram att m2 inte rör sig lika fort mot m3(m2 mot m3 + m3 mot m2) som m1 mot m3 (m1 mot m3 +m3 mot m1). Om vi nu säger att m3 är jorden så stämmer inte galileis påstående (detta under förutsättning att man kan räkna på ovan beskrivet sätt). Jag har grubblat länge på det här. Min fråga lyder åter igen: stämmer Gallileis påstående? eller är det Newtons mekanikformler som inte stämmer (jag har även räknat med Einsteins speciella relativitetsteori) eller är det ett matematiskt problem? mvh Rafi Sheik
/Rafi S, Säveskolan, Visby Gotland

Svar:
För att vara konkreta tänker vi oss att alla tre partiklarna ligger på ett biljardbord. All acceleration räknas relativt bordet (som ligger still).

Kropp 3 får då en acceleration mot kropp 2 men även mot kropp 3. Adderar du dessa accelerationer enligt regler för vektoraddition blir det inga motsägelser.


/GO

*

Blandat [1298]

Fråga:
Hej, skulle vilja veta lite mer om vad tid är, och varför tiden skulle gå saktare om man reser snabbt. Framförallt är jag osäker på vad som menas med tid? Har ni några litteraturtips eller dyl. vore jag tacksam.
/Magnus H, Rudbecksgymnasiet, Tidaholm

Svar:
Tid är ett grundbegrepp som inte går att definiera utgående från andra grundläggande storheter. Detta var en av Einsteins utgångspunkter när han formulerade relativitetsteorin. Han gjorde en så kallad operationell definition: Tid är det som mäts med klockor. Med denna "definition" kan man ge begreppet tid ett fysikaliskt innehåll. Det gäller bara att vi inom fysiken kommer överens om vad som menas med en klocka.

Läs: "Upptäck Fysik B" av L Jakobsson och G Ohlén (Gleerups förlag). På sidorna 292 och framåt diskuteras tidsbegreppet utifrån ovanstående utgångspunkt.

I länk 1 finns den moderna definitionen på enheten för tid - sekund.


/GO/lpe

Se även fråga 1295

Nyckelord: tid [10];

1 https://physics.nist.gov/cuu/Units/second.html

*

Universum-Solen-Planeterna [1328]

Fråga:
Vad innebär det att universum är ett krökt rum?
/Eva-Lena L, Mönsteråsgymnasiet, Mönsterås

Svar:
Det krökta rummet är ett sätt att beskriva gravitationspåverkan.

Enligt Newtons teori så påverkas t ex jorden av en kraft från solen. Denna kraft gör att jordens bana avböjes, den går inte rakt fram utan följer en nästan cirkelformig bana.

Enligt Einsteins gravitationsteori (den allmänna relativitetsteorin) så gör stora massor (t ex solen) att rummet blir krökt. Jorden påverkas då inte av en kraft utan går "rakt fram" i ett krökt rum. Banan blir böjd eftersom rummet i sig själv är krökt.

Fundera Tänk på en myra som kryper på ett äpple. Trots att den själv tycker att den kryper rakt fram så går den i en krökt bana. Den kan t ex gå i en cirkel runt äpplets skaft.

Test Hur vet man att rummet är krökt? Ett sätt är att se om parallella linjer möts eller går isär. Ta fram en jordgob och dra två räta parallella linjer. Möts de?

Kommentar Eftersom gravitationskraften på de flesta ställen är svag är rummets krökning väldigt liten utom nära svarta hål.
/GO

*

Blandat [1347]

Fråga:
Vad är eterns densitet?
/Veronica F, hangö högstadium, hangö - finlandd

Svar:
Enligt alla moderna fysikaliska teorier finns det ingen eter, så den har ingen densitet. Det var Einstein som med sin relativitetsteori gjorde eterbegreppet överflödigt i fysiken.


/GO 

*

Ljud-Ljus-Vågor [1400]

Fråga:
Läste i Illustrerad vetenskap nr 5-98 om ett rymdteleskop med solen som lins. Hur ser formeln ut?
/björn w, Karlbergsgymnasiet, Åmål

Svar:
Tanken är att utnyttja solen som gravitationslins. Ljuset påverkas av gravitationen (tyngdkraften) så att det böjs av. En ljusstråle som passerar nära solranden böjs av 1.74 bågsekunder (1 bågsekund = 1/3600 grad) enligt Einsteins allmäna relativitetsteori. (Den teorin beskriver egentligen rummet som krökt och att ljuset går rakt i detta rum.) Parallella strålar som passerar nära solranden kommer då att brytas samman till en "brännpunkt" som ligger ganska långt bort, 550 AE (1 AE = jordbanans radie = 150 miljoner km). Härifrån skulle man se himlakroppar bakom solen förstorade. Förstoringen blir inte särskilt stor, och svårigheterna att genomföra det är oerhörda. Artikeln ger en helt orealistisk bild av vad som kan åstadkommas.

Gravtationslinser är i alla fall i praktiken påvisade, och när det gäller stora massor, som galaxhopar kan fenomenet vara påtagligt. Hubble-teleskopet har tagit flera bilder där man ser bakomliggande galaxer förstorade och förvrängda av galaxhopens gravitation. Med hjälp av dessa bilder kan man "väga" hopen och beräkna hur materien är fördelad. I ett fall ser man samma galax avbildad på 5 olika ställen. Du kan titta på den på länk 1 och bilden nedan. Läs gärna den beskrivande texten.

Räkna ut solens "brännvidd" som gravitationslins. Solradien är 0.7 miljoner km. Övriga uppgifter du behöver finns i detta svar.

Svar: 480 AE. I artikeln anges 550 AE. Man har kanske räknat ut det på ett lite annorlunda sätt. Ska man göra det riktigt, ska man ta med ljus som passerar lite längre från solen, och inte bara strålarna som går precis vid solranden.



/KS/lpe

Se även fråga 1328

Nyckelord: gravitationslins [5]; Illustrerad Vetenskap [17]; relativitetsteorin, allmänna [33];

1 http://hubblesite.org/newscenter/newsdesk/archive/releases/1996/10/

*

Universum-Solen-Planeterna [1409]

Fråga:
Om man befinner sig i en sluten låda där man märker att man pressas mot en av lådans väggar, kan man då avgöra (praktiskt och/eller teoretiskt) om lådan befinner sig i en accelererande raket eller om den står på jorden?
/Staffan W, Åva, STOCKHOLM

Svar:
Nej! Detta tänkta experiment var en av utgångspunkterna för Einstein när han formulerade sin allmänna relativitetsteori. Hans grundläggande antagande var: Det finns ingen fysikalisk skillnad mellan ett gravitationsfält och ett accelererande system.


/GO

*

Energi [1459]

Fråga:
Jo jag undrar en sak... Enstein sa det att E=MC2 (tyvär klarar jag inte ha 2an upphöjd). Av detta kan man tyda att materia kan omvandlas till energi. Men skulle man inte då kunna omvandla energi till materia? (jag som spånar) Kan man inte då fånga upp ljus energi och om vandla det till låt oss säg väte atomer? Sen var det det också att vi har alla lärt oss att ljus, värme, rörelse, läges är energier och att de inte kan förstöras eller skapas bara omvandlas. Om man då omvandlar någon energiform till materia borde materia vara en energi form det också. Och all materia har en massa, och allt som har en massa har en dragningskraft (tror jag bestämt att jag har lärt mej), då borde gravitation också vara en energiform.?!? Och sist: Jag läser just nu om relativitets teorierna och vill gärna ha hjälp med att hitta någon "sajt" med enkel och lättförsåelig fakta om detta, finns det? //tack, Said
/Said A, Fäladsgården, Lund

Svar:
Einsteins berömda formel (E=mc2) säger just att energi och massa är likvärdiga. Energi har alltså massa, liksom materia. Ljus kan i princip omvandlas till materia, men för synligt ljus går det nog inte att hitta något bra exempel. Har vi i stället ljus (eller rättare sagt elektromagnetisk strålning) med mycket kortare våglängd (högre energi), kan denna strålning mycket väl bilda en elektron och en anti-elektron. Alltså materia! Anti-elektronen kommer så småningom (i själva verket går det väldigt fort) i närheten av en annan elektron, och de förintar varandra. Materien har övergått i strålning.

Med tillräckligt hög energi hos strålningen kan en vätekärna och en anti-vätekärna bildas.

Gravitationen har också energi, och därmed massa. Det mest drastiska exemplet på detta är ett svart hål, som kan vara väldigt tungt. Hela massan ligger här i gravitationen!
/KS/lpe

*

Materiens innersta-Atomer-Kärnor [1513]

Fråga:
Hejsan! Satt och tänkte på Einsteins berömda uttalande gällande kvantmekaniken, som han ju satte sig emot. Hade han ändå inte rätt i grund och botten då han sade att "Gud kastar inte tärning med oss"? För om jag förstått saken rätt så går hela kvantmekaniken ut på en gräns, från vilken vi inte kan fastställa både en partikels läge och dess hastighet exakt. Vi är oförmögna att göra det, men det behöver väl inte betyda att den inte har exakta värden på läge och hastighet?
/Nina , Norreport, Ystad

Svar:
Här kommer du in på mycket spännande och grundläggande frågor, svåra också. Det du beskriver kallas Heisenbergs obestämbarhetsrelation, och den har varit under diskussion med ungefär de resonemang som du tar upp. Alltså, det är en sak vad vi kan mäta, och en annan om det finns en "exakt" verklighet bakom.

Slumpmässigheten i kvantmekaniken är faktiskt mycket mer fundamental. Man har på senare år kommit underfund med, att alla delar av universum, i en viss mening, tycks stå i ögonblicklig kontakt med varandra. Den som kom på det hette John Bell.

Det där låter som rena nippran, inget kan ju gå fortare än ljuset! Det som inte kan transporteras snabbare än ljuset är energi och information. Vissa fysiker tror, att vi här kan få en djupare förståelse för slumpmässigheten i kvantmekaniken. Den tycks behövas för vi inte ska kunna skicka information ögonblickligen. Om det skulle ha varit möjligt, skulle universum ha sett helt annorlunda ut. Det här går nog inte att förklara så att du förstår allt. Kanske ingen kan förstå det riktigt.

Det finns ett annat exempel på denna gåtfulla fjärrkontakt, som varit känd långt före kvantmekanikens tillkomst: Machs princip. Det finns ingen absolut rörelse, men det finns absolut rotation. Sitter vi i en roterande ihålig kula och inte kan se ut, kan vi inte avgöra om den rör sig, men vi märker att kulan roterar. Vi känner av centrifugalkraften. Vi måste alltså på något vis stå i kontakt med något utanför.

I den engelska tidskriften New Scientist, 22 augusti 1998, finns en artikel, som tar upp dessa svåra, men intressanta frågor. Den har just titeln "Why God plays dice" eller, på svenska "Varför Gud kastar tärning" (se bilden nedan).

Tillägg 2/3/05 (Peter E):
Som Kaj säger är detta en mycket djup och svårförståelig aspekt av kvantmekaniken. Under länk 1 nedan finns länkar till Einstein-Podolsky-Rosen (EPR) artikeln, Bells artikel och en beskriving av Aspects experiment. Bell's Theorem är en relativt svår artikel som emellertid innehåller en länk till en förenklad analogi (länk 2 nedan). En varning dock igen: det är inte enkelt!

Sammanfattningsvis kan man säga att Aspects experiment (och flera senare) till 100% bekräftar kvantmekaniken. Vi får acceptera slumpmässigheten och det tycks inte vara något problem med relativitetsteorin eftersom man inte kan få fotonerna i försöket att överföra någon information.

Se även EPR_paradox .



/KS/lpe

Se även fråga 951

Nyckelord: EPR, Bell, Aspect [3];

1 http://www.drchinese.com/David/EPR_Bell_Aspect.htm
2 http://www.upscale.utoronto.ca/GeneralInterest/Harrison/BellsTheorem/Analogy.html

*

Kraft-Rörelse [1566]

Fråga:
Vad är einsteinsrelativitets teori baserad på? / vill ha långt svar
/Johan W, Berga

Svar:
Du får ett långt svar, men inte här. Läs först artikeln på uppslagsordet relativitetsteori i Nationalencyklopedin . Den är bra. Läs sedan frågorna och svaren på denna databas. Det är många som grunnar på dessa saker. Sedan är du välkommen med nya problem!

Här finns mer om Einstein: Albert Einstein
/KS/lpe

Avancerad sökning på 'Einstein relativitet' i denna databas

*

Ljud-Ljus-Vågor [1618]

Fråga:
Hej! Vad jag förstått är gravitation en direkt verkande kraft. När gravitation verkar över stora avstånd tex i rymden tar det då tid för gravitationen att nå fram, den har väl ingen hastighet utan verkar direkt på alla kroppar. Om så är fallet skulle man väl märka gravitationsstrålningen vid sammansmältandet av neutronsjärnor innan man kan observera det (det tar tid för ljuset att nå fram). Om inte måste väl gravitationen också verka över tiden(ha en hastighet). (Jag förstår inte riktigt skillnaden mellan gravitation och gravitationsstrålning!)

Min fråga är verkar gravitationsstrålning över tiden (sen big bang brukar det stå), om den gör det har den en hastighet?
/Jonas T, Hemskolan, Linöping

Svar:
Gravitationsstrålningen spelar samma roll för gravitationen som ljuset inom elektromagnetismen. I Einsteins allmänna relativitetsteori går den med samma hastighet som ljuset, men det finns andra teorier, där den är något långsammare. Einsteins teori är ju väletablerad, så de flesta tror nog att de går med ljusets hastighet. I så fall skulle gravitationsstrålningen och ljuset nå oss samtidigt från en kosmisk katastrof.

Än har ingen påvisat strålningen, men det byggs flera jättelika anläggningar, till exempel Laser Interferometer Gravitational Wave Observatory (LIGO) .
/KS/lpe

Nyckelord: gravitationsvågor [19];

*

Kraft-Rörelse [1694]

Fråga:
Enligt Einstein så kan inte information skickas snabbare än ljusets hastighet, men om man skulle dra ett snöre från Mars till Jorden, och sedan rycka i snöret vid Jorden så borde väl snöret röra på sig vid mars vid exakt samma tillfälle som man drog i det på Jorden. Om det går att göra så så skulle man teoretiskt kunna rycka i snöret och genom morsekod telegrafera ett meddeland snabbare än ljuset.
/John S, Luleå, Luleå

Svar:
Om du rycker i snöret, fortplantar sig rycket med ljudets hastighet, som säkert är under 1000 m/s. Snörkommunikation är alltså mycket långsam. Stela kroppar finns inte enligt relativitetsteorin.

Räkna ut: Hur lång tid tar det för rycket att nå Mars, om avståndet till Mars är 200 miljoner km, och ljudhastigheten är 1 km/s?  
/KS

Se även fråga 1519 och fråga 817

*

Kraft-Rörelse [2374]

Fråga:
Att det tar en tid för ljuset att ta sig från en punkt till en annan är ju allmänt känt men hur är det med gravitationen? Jag undrar alltså: Hur lång tid tar det för en kropp att påverkas av av en annan kropps gravitationfält? Exempel:Om man hypotetiskt skulle säga att solen upphörde att existera, skulle man då märka det på jordens rotationsbana innan man upptäckte att det blev mörkt.
/Sven J, Täby

Svar:
Gravitationen utbreder sig enligt Einsteins allmänna relativitetsteori med ljusets hastighet. Om t ex en stjärna exploderar (supernova) så sänds det ut gravitationsvågor som rör sig med ljusets fart.

Om man kunde tänka sig att solen (och dess massa) plötsligt försvann så skulle inte gravitationskraften försvinna förrän efter 8 minuter.
/GO/lpe

Nyckelord: relativitetsteorin, allmänna [33];

*

[2379]

Fråga:
Om man rör sig snabbare än ljuset kommer då tiden att gå bakåt! (Einsteins relativitetsteorin)
/Henric K, Tranås

Svar:
Frågan är ofysikalisk. Man kan inte röra sig fortare än ljuset.
/GO

*

Ljud-Ljus-Vågor [2453]

Fråga:
Jag undrar var jag kan hitta information om Albert Einsteins uppfinningar inom optik. Även vad Newton gjorde för optiken. Var kan jag hitta detta? Eller kan ni svara?
/Sara S, Duveholmsskolan, Katrineholm

Svar:
Einstein fick sitt nobelpris (1921) för sin analys av den fotoelektriska effekten, inte för relativitetsteorin, konstigt nog. Här kan du hitta mera information: The Nobel Prize in Physics - Laureates .

Newton sysslade bland annat med färglära. Han byggde det första spegelteleskopet. Båda kan du läsa om i Nationalencyklopedin . Där står en hel del. 
/KS/lpe

*

Materiens innersta-Atomer-Kärnor [2559]

Fråga:
Jag läste i eran mycket bra frågelåda om ljus och mörker, och där skrev ni att fotoner (ljuspartiklar) inte har någon massa. Men i Einsteins relativitetsteori står det E=MC2 energi lika med massa och ljus är energi eller är jag på fel spår? Tack på förhand.
/Fredrik O, Ribbyskolan, Västerhaninge

Svar:
Den har ingen vilomassa. En partikel som far med ljushastigheten kan, enligt relativitetsteorin, inte ha någon vilomassa. Det betyder att en foton kan ha hur liten energi som helst. Ljusets energi (E) är relaterad till frekvensen (f) enligt:

E = f * h

där h är Plancks konstant. Någon vilomassa kan vi inte blanda in. Ändå påverkas ljuset av gravitationsfält så att det böjs av.
/KS

Se även fråga 1400

*

Kraft-Rörelse [2697]

Fråga:
Jo, jag ska hålla ett litet föredrag om RELATIVITETSTEORIN. Ett ämne som kanske inte är det lättaste. Jag har samlat lagom med teoretisk fakta, men skulle vilja ha ett experiment som grädde på toppen. Har ni förslag på hur man kan visa att tidsdilatationen verkligen fungerar?
/Markus A, Komvux, Sjöbo, Sjöbo

Svar:
Kanske inte ett experiment, men ett bra exempel.

Först ett par definitioner:

Längdkontraktion är den minskning i längd som enligt Albert Einsteins speciella relativititetsteori uppstår när ett föremål rör sig med stor hastighet i förhållande till den som mäter längden. Vid mer vardagsnära hastigheter är denna längdminskning helt försumbar. Det är först vid hastigheter som är minst 1/10 av ljusets hastighet som den får någon märkbar betydelse.

Tidsdilatation (tidsutvidgning) beroende på hastighet innebär att om två referenssystem r och r', har identiska klockor, kommer en observatör i r att anse att klockan i r' går långsammare om referenssystemen r och r' befinner sig i relativ rörelse. En observatör i r' anser likaså att klockan i r går långsammare än den lokala klockan.

Kosmiska strålningen består huvudsakligen av atomkärnor med mycket höga energier (mest väte). När de kolliderar med luften på cirka 20 km höjd, uppstår en uppsjö av olika partiklar. De flesta är kortlivade och sönderfaller snabbt. En av sönderfallsprodukterna kallas myon, och på grund av sina egenskaper, är den mycket lite benägen att kollidera med kärnorna i luften. Den försvinner för det mesta genom att den sönderfaller. Hur långt går den?

Myonens medellivslängd är 2.2*10-6 s. Antag att den går nära ljushastigheten (300000 km/s). Då får vi en sträcka på

300000 * 2.2*10-6 = 0.6 km = 660 m.

Ändå är det så, att de flesta partiklar vi kan registrera här nere är myoner. Genom en människa far det typiskt 30 - 40 högenergetiska myoner varje sekund. De överlever hit ner på grund av tidsdilatationen. De har så hög hastighet att tiden går mycket långsammare för dem, se översta formeln i bilden nedan. En myon med en Lorentz-faktor på 1000 kan i princip gå 660 km. Sådana myoner är inte alls ovanliga. För den myonen går tiden 1000 gånger långsammare (från oss sett).

Jaha, OK men sett från myonen då? Där går ju tiden med "normal" hastighet och myonen kan väl inte hinna ner till markytan innan den sönderfaller? Jo, det gör den därför att längdkontraktionen, se nedersta formeln nedan, gör att den sträcka myonen måste tillryggalägga är mycket kortare (sett ur myonens perspektiv).

Det är klart att resultatet att myonen hinner ner till marken innan den sönderfaller måste vara samma oberoende av om vi betraktar myonen från marken eller om vi följer med den ner genom luften.

Se vidare Muon , Time_dilation och Length_contraction .



/KS/lpe

Se även fråga 1289 och fråga 2627

Nyckelord: relativitetsteorin, speciella [45]; längdkontraktion [6]; tidsdilatation [6];

*

Kraft-Rörelse [2930]

Fråga:
Hur kan man undersöka vad som skiljer Einsteins teorier från den klassiska beskrivningen av elektromagnetisk strålning?
/Simon *, ***, ***

Svar:
I den speciella relativitetsteorin utgår man från två postulat*).

1. Fysikens lagar är lika i alla system i likformig rörelse.

2. Ljusets hastighet i vakuum är alltid densamma.

Konsekvent tillämpat leder detta till den speciella relativitetsteorin. I synnerhet det sista är svårt att förena med vardagserfarenheten.

*) Ett postulat är ett påstående som inte behöver (och inte kan) bevisas.
/KS

Se även fråga 1792 och fråga 1523

*

Universum-Solen-Planeterna [3005]

Fråga:
Vi har några frågor angående universums utvidgning. 1. Hur kan något som kallas oändligt och är utan mittpunkt utvidgas? 2. Hur räknades den kosmologiska konstanten ut? 3. Är Big crunch teorin helt förkastad, hur stark har den varit? 4. Ungefär med vilken hastighet utvidgar sig universum? 5. Hur säkra är de att teorin om universums utvidgning stämmer, sättet att mäta super novor? 6. Finns det några utarbetade teorier om att det finns andra universum? 7. Bär universum skapades och började det utvidgas snabare än ljuset. Det sägs att så mycket energi inte finns, kan det vara möjligt att den energin funnits men sedan omvandlats till materia? 8. Hur kan materia bildats ur "ingenting"?
/Emilia B, Porthälla Gymnasium, Sävedalen

Svar:
Vi ska börja med den kosmologiska konstanten, som vi tydligen inte tagit upp tidigare. Einstein var tidigt klar över att fältekvationerna i den allmänna relativitetsteorin inte tillåter ett statiskt (oföränderligt) universum. Expansionen av universum var inte upptäckt då. Därför införde han 1917 en ny term i en av ekvationerna, den kosmologoska konstanten (brukar betecknas med l). Fenomenet skulle motverka gravitationen på stora avstånd. När sedan expansionen upptäcktes, tyckte Einstein att han hade gjort en dundertabbe med den där konstanten. Nu har man på senare år fått fram data, som tyder på att universums expansion accelererar. I det sammanhanget har den kosmologiska konstanten blivit aktuell igen. Se svaret på fråga nedan.

I övrigt hittar du nog svaren i denna databas. Sök till exempel på Big Bang och Big Crunch och Hubblekonstanten och supernovor och andra universa.

Är du inte nöjd med vad du hittar, är du välkommen åter!  
/KS

Se även fråga 2561

*

Kraft-Rörelse [3061]

Fråga:
En fråga har dykt opp om relativitetsteori: Observatör A står stilla på marken vid en järnvägstunnel. Observatör B åker i ett tåg med samma vilolängd som tunneln. Tåget rör sig relativt A med en hastighet nära ljusets. På rälsen finns två strömbrytare, en alldeles före och en alldeles efter tunneln. En sådan trycks ner om någon del av tåget befinner sig ovanför (alltså inte bara när själva hjulen passerar). Strömbrytarna är seriekopplade i en krets med ett batteri och en lampa som alltså lyser bara då båda strömbrytarna är nertryckta. P g a längdkontraktion tycker B att tåget är längre än tunneln och att båda strömbrytarna därför är nertryckta samtidigt en liten stund. Lampan torde alltså lysa för honom. För A däremot är tåget kortare än tunneln och lampan lyser inte! Men om lampan lyser för den ene måste den lysa för den andre också - det är ju invariant och oberoende av observatör. Kan du hitta felet i resonemanget? Lyser lampan?
/Jonas S, Spånga gymnasium, Spånga

Svar:
Haken i resonemanget är det där lilla ordet samtidigt, som man måste vara mycket noga med hur man använder i relativitetsteorin. Det var just med en analys av detta begrepp, som Einstein inledde sin berömda artikel från 1905, se fråga 12753 . Man kan inte använda det ordet som vi är vana i vardagstillvaron.

Signalerna från brytarna måste ju transporteras till lampan, och det kan ju inte gå fortare än ljuset. Beroende på vilken transporthastighet man antar, tågets hastighet och lampans placering får man olika resultat. Det går i varje fall att analysera situationen, så att resultatet blir det samma, sett från tunneln och tåget.

Tillägg 21/4/08:

Länk 1 nedan 'The Train and The Twins' diskuterar tågparadoxen (ytligt) och tvillingparadoxen ingående. Länk 2 (Storrs McCall & E. J. Lowe: 3D/4D equivalence, the twins paradox and absolute time) säger att tågparadoxen endast kan lösas om man betraktar problemet i full rymd-tid 4D. Problemet är en perspektiveffekt. Se även diskussion i Taylor/Wheeler: Spacetime Physics. Det finns för övrigt flera varianter av tåg-tunnelparadoxen: en linjal och lucka i bordet, en hoppstav och en lada mm.
/KS/lpe

Nyckelord: relativitetsteorin, speciella [45]; tvillingparadoxen [5];

1 http://galileoandeinstein.physics.virginia.edu/lectures/sreltwins.html
2 http://www.mcgill.ca/files/philosophy/Analysis.pdf

*

Kraft-Rörelse [3465]

Fråga:
Det finns ju två förklaringar på gravitationen, Einstein och Newton. Man räknar ofta med Newtons räkneregler i skolan innebär det att det är hans teori som är allmänt accepterad eller är det så att Einsteins formel är den rätta men den är så svår att räkna med.
/David S, Åbo Akademi, Åbo Finland

Svar:
Man kan säga att Newtons gravitationsteori är en mycket god approximation av Einsteins teori, så länge vi inte har extremt starka gravitationsfält, eller kräver extremt hög noggrannhet. Det är inte bara i skolorna som man räknar med Newtons teori. Det gör också rymdfolket som skickar sonder till Mars. Det här är ett bra exempel på att man inte i fysiken ska tala om "rätt" eller "fel" teori, utan snarare om modeller, som är mer eller mindre användbara i olika sammanhang. Sedan är det så, att Einsteins gravitationsteori (allmäna relativitetsteorin) är mycket svårare att räkna med, så den använder man bara när det verkligen är nödvändigt.

Rymdfolket måste ta hänsyn till den speciella relativitetsteorin, men det är ju inte någon gravitationsteori. För extremt precisionskänsliga tillämpningar, t.ex. GPS, måste man ta hänsin även till den generella gravitationsteorin, se fråga 14685 nedan.
/KS/lpe

Se även fråga 14685

Nyckelord: Newtons gravitationslag [12];

*

Partiklar [3491]

Fråga:
Jag har hört att en foton som ändrar spin samtidigt ändrar "tvillingfotonens" spin. Min lärare har sagt att det ligger till så här, men inte kunnat ge någon förklaring på hur det går ihop med relativitetsteorierna som säger att ingenting kan färdas snabbare än ljuset (inklusive information om spintal)
/Lars M, Angeredsgymnasiet, Angered

Svar:
Nu är det så att fotonen alltid har spin 1, så det kan inte ändras. Men vi begriper att det är ett annat problem du tar upp. Det går tillbaka till år 1935, då Albert Einstein, Boris Podolsky och Nathan Rosen publicerade en artikel, som ledde till intensiva diskusioner om den så kallade EPR-paradoxen. Utgångspunkten var ett tankeexperiment där en partikel sönderfaller till två partiklar, den ena med spin upp och den andra med spin ner. Enligt en tolkning av kvantmekaniken är tillståndet obestämt ända tills man mäter vilket spin den ena partikeln har. Då blir den andra partikelns spin omedelbart också bestämt, ovsett hur långt borta den är. Det var framför allt Niels Bohr som argumenterade för detta. Einsten kunde inte acceptera denna tolkning, han menade att på något sätt måste det vara förutbestämt vilket spin de båda partiklarna ska ha, kanske i form av "dolda variabler" som vi ännu inte har upptäckt.

I många år handlade det enbart om tankeexperiment och filosofiska argument. År 1969 publicerade John Bell en artikel, som helt vände upp och ner på läget. Han anvisade en metod där saken skulle kunna avgöras med experiment. Sedan dess har åtskilliga experiment gjorts. Man har använt fotoner, och den mätta parametern är inte spin, utan polarisering. Experimenten är inte lätta, men de har förbättrats genom åren. Nu verkar resultaten vara entydiga. Bohr hade rätt, Einstein hade fel.

Kvantmekaniken är verkligen fantastisk. De båda partiklarna står i kontakt med varandra, oavsett avstånd och tid. Och den kontakten är ögonblicklig. Det tycks strida mot att inget kan gå fortare än ljuset. Här har vi kanske en djupare förklaring till slumpmässigheten i kvantmekaniken, som nämligen garanterar att vi inte kan överföra information med detta fenomen. Om det hade varit möjligt att momentant överföra information, skulle universum ha varit helt annorlunda.  
/KS

Se även fråga 1513

Nyckelord: EPR, Bell, Aspect [3];

1 https://www.lcf.institutoptique.fr/content/download/3309/22271/file/2004%20Bell%20thorem%20naive%20view%20arxiv%20quant%20ph%200402001.pdf

*

Kraft-Rörelse [3581]

Fråga:
Hej! Jag undrar hur Einstein kunde utgå från att ljushastigheten är konstant! Hela hans resonemang bygger på det antagandet... Men om den inte _är_ det, då? Då faller hans teorier, men de är kanske fel ändå i så fall?
/Anders J, Katedralskolan, Lund

Svar:
Redan år 1887 hade Michelson och Morley publicerat resultaten av experiment, som måste tolkas som att ljushastigheten är konstant. Einsteins antagande var alltså inte fria fantasier. Han drog helt enkelt konsekvenserna av nästan 20 år gamla experimentella resultat. Den speciella relativitetsteorin publicerades år 1905 i Annalen der Physik.
/KS

Se även fråga 3545 och fråga 2930

*

Ljud-Ljus-Vågor [3623]

Fråga:
Jag har läst att Einstein gjorde en uppsats där han kom fram till de Broglies formel. Hur kom han fram till den formeln?
/Rob K, Högberget, Ludvika

Svar:
Var har du läst det? Einsteins viktigaste arbeten, den speciella och den allmänna relativitetsteorin publicerades 1905 och 1917 respektive. de Broglie publicerade sin vågekvation år 1924 i sin doktorsavhandling. Han gjorde där det djärva antagandet att varje partikel också hade vågnatur. Han "gissade" att den mekaniska rörelsemängden (mv) var lika med rörelsemängden för motsvarande våg (h/l). Det ger de Broglies vågekvation:

l = h/mv

l = våglängden, h = Plancks konstant, m = massan, v = hastigheten.

3 år senare påvisades effekten experimentellt. Allt stämde! År 1929 fick han nobelpriset för sin insats, se The Nobel Prize in Physics - Laureates . Nog var det de Broglie som kom på det själv! 
/KS/lpe

*

Universum-Solen-Planeterna [3810]

Fråga:
Angående universums utvidgning så har jag studerat och tagit till mitt hjärta modellen av en ballong som blåses upp, där den tvådimensionella ytan utvidgar sig i en tredimensionell omgivning i analogi med att det tredimensionella rumsuniversum utvidgar sig i en fyrdimensionell omgivande struktur, alltmedan masspartiklarna behåller sin rumsliga storlek. När vi kom in på detta som ett sidospår då vi i klassen diskuterade relativitet, var det en elev som undrade om man inte lika gärna kunde betrakta det som att universum hade sin konstanta rumsliga utsträckning och det i själva verket var galaxerna och deras innehåll som minskade i storlek, relativt universum. Jag kunde inte hitta något fel i hans resonemang, under förutsättning att naturlagarna kontinuerligt modifieras efter den nya storleken. Alla effekter kring universums utvidgning (och även kring Big Bang) torde bli samma enligt denna modell. Eller var finns fallgropen?
/Fredrik M, Polhemsgymnasiet, Göteborg

Svar:
Man få komma ihåg att en fysikalisk teori (eller modell) aldrig ger sig ut för innehålla sanningar. Modellen beskriver verkligheten, bättre eller sämre. Det är till exempel en missuppfattning att säga att Newton hade fel och Einstein hade rätt. NASA använder Newtons mekanik och gravitationsteori för sina rymdsonder.

Visst kan man ställa upp en modell som du beskriver, men den kan avvisas enligt en princip, som formulerades reda på 1300-talet.

Ockhams rakkniv innebär att man bör inte anta existensen av flera ting eller företeelser än nödvändigt för att förstå eller förklara de ifrågavarande - konkreta iakttagna - fenomenen. Det vill säga: ju enklare desto bättre! (se Ockhams_rakkniv )

Det har gjorts många undersökningar om naturkonstanterna ändras med tiden. Ingen har funnit något tecken på detta.

Den här "förminskningsmodellen" ger ingen förklaring till tidsutvecklingen. För 10000 miljoner år sedan var kvasarer vanliga, nu finns knappast några. Inte heller 3-gradersstrålningen går att begripa.

Men det är jättekul att ni funderar på dessa saker. Och att ifrågasätta är mycket viktigt!
/KS/lpe

*

Kraft-Rörelse [3811]

Fråga:
Einsteins teorier Om jag knallar runt i en cirkel på ca 2 meters diameter så gör jag det på några sekunder! Men man skulle ju lika gärna kunna se det som att resten av världen har cirklat runt mig på samma tid eftersom rörelser är relativ. Då skulle det innebära att stjärnor på ljusårs avstånd har färdats i en enorm cirkel runt mig på några sekunder och det betyder att dom måste ha haft en hastighet långt över ljushastigheten! Hur går det här ihop med Einsteins teorier egentligen? Kristjan
/Kristjan S, Går inte i nå'n skola!, Vad för ort?

Svar:
Varken i Newtons eller Einsteins mekanik finns absolut rörelse. I den meningen kan båda kallas relativitetsteorier. Däremot finns absolut rotation, det går att avgöra om ett system roterar och hur fort det snurrar. Man behöver inte titta på stjärnhimlen, det räcker att mäta centrifugalkraften. Ibland kallas detta Mach's pricip. Se vidare länk 1.
/KS

Se även fråga 1513

1 http://en.wikipedia.org/wiki/Mach's_principle

*

Ljud-Ljus-Vågor [4276]

Fråga:
Hur kan ljuset vars hastighet är konstant uppvisa dopplereffekt? Ljus som "segar" sig ut från ett gravitationsfällt rödförskjuts, medan ljus som "ramlar" in i ett, blåförskjuts (rätta mig ifall jag fattat fel ). Har de båda olika ljusstrålarna olika hastighet?, vilken annan energi än rörelseenergi skulle annars vara större hos det "infallande", blåförskjutna ljusknippet än hos det som "segar" sig ut? Tack på förhand / Pär
/Pär F, Berzan, linköping

Svar:
Einsteins antagande är att ljushastigheten är konstant i vacuum, oberoende av hur man rör sig. Einsteins storhet var att han drog de strikta konsekvenserna av detta ganska egendomliga antagande. Det ledde till den speciella relativitetsteorin. Där är det inga problem med dopplereffekten.

Man kan också förklara det med att klockan ser ut att gå långsammare i ett system med rödförskjutning. Du kan tänka dig ljusvågorna som klockpulser.

Ska man diskutera röd- och blåförskjutning i gravitationsfält får man blanda in den allmänna relativitetsteorin, som ju är en gravitationsteori.
/KS

*

Kraft-Rörelse [4828]

Fråga:
Jag vet att tvillingparadoxen förklaras med att de båda tvillingarnas resor inte är identiska pga acceleration och inbromsning. Men hur förklaras detta matematiskt? I formlen för tidsdilatation tas ju ingen hänsyn till detta.
/Thomas N, Gullstrandskolan, Landskrona

Svar:
Man kan säga att tvillingparadoxen uppstår därför att Einstein inte var tillräckligt radikal matematiskt sett i sin ursprungliga formulering av den speciella relativitetsteorin. I Minkowskis formulering uppkommer ingen paradox. Man räknar ut egentiden genom att sätta in siffrorna i en formel. Det blir helt enkelt olika egentider för de båda tvillingarna.

Sök på tvillingparadoxen i denna databas!

I länken nedan finns en trevlig Java-applet och mer information.
/KS/lpe

1 http://www.pbs.org/wgbh/nova/einstein/hotsciencetwin/twin1.html

*

Universum-Solen-Planeterna [5067]

Fråga:
Hej!

Jag har en fråga ang mitt specialarbete om Svarta hål:

Rörelsemängdsmomentet för en kropp är alltid konstant. När en stjärna blir till ett svart hål, minskar radien, vilket innebär att rörelsemängden måste öka. Eftersom vmax= c måste m gå mot oändligheten. Stämmer detta? Samtidigt förlorar hålet energi (som ju är identiskt med massa enl Einstein) vid komprimeringen. Hur går detta ihop?

Tacksam för svar!
/Karl Johan H, Katedral, Växjö

Svar:
Tyvärr, sådana resonemang som grundas på vardagsfysik blir fel nära svarta hål. Exempelvis energiprincipen (lagen om energins bevarande) gäller inte här. Problemet måste angripas med allmän relativitetsteori, och det är långt utanför gymnasiekursen. Har du prövat att söka på svart hål eller svarta hål i denna databas?
/KS

*

Universum-Solen-Planeterna [5537]

Fråga:
I fredags läste jag en artikel i sydsvenskan om att forskare har bevisat att rymden är platt. Det stod också att detta bevisar att ljuset går i raka linjer och inte runda. Att universum är platt förstår jag men ljuset och rumstiden kan väl fortfarande krökas av gravitation från materia? Om jag inte minns fel har detta också redan visats när man vid solförmörkelser har observerat att stjärnor har fått nya platser på himmelen p.g.a att ljuset böjs av solens gravitation
/Klas o

Svar:
Du har alldeles rätt. Ljuset böjs i gravitationsfält, det är väl belagt. Det där med att "Einstein hade fel" har artikelförfattaren fullkomligt missförstått. Artikeln som referas till i sydsvenskan finns i Nature Vol 404, 27 April, 2000, sid 955. Vad man studerar är små ojämnheter i den kosmiska bakgrundsstrålningen (2.7 K strålningen). Av dessa mätningar får man fram att rummets krökning är mycket liten. För att komma fram till denna tolkning, använder man sig av Einsteins allmänna relativitetsteori.
/KS

Se även fråga 228

*

Universum-Solen-Planeterna [6025]

Fråga:
Ett klipp från mitt specialarbete:

" En sak som jag undrar över varför ingen har nämnt är det relativa gravitationsfältet. Både Maxwell och Einstein konstaterade ju att alla kraftfält så som t ex gravitationen utbreder sig med ljusets hastighet. Om en stjärna exploderar så upphör inte dess gravitationsfält omedelbart utan gravitationen försvinner sakta utåt med ljusets hastighet. Det jag undrade var att varför skulle inte den här hastigheten vara lika konstant som ljuset med avseende på rörelse i förhållande till iakttagaren? Det skulle ju innebära att man skulle kunna "se" hur en massiv kropp som rör sig i hög hastighet släpade sitt gravitationsfält efter sig. Men det har jag inte hört någon nämna något om. "

Varför står det inte någonstans om detta ? Har letat efter ett svar nu i ett halvår men inte lyckats hitta nåt.... Är jag helt ute och cyklar eller ?
/Martin S, Malmö

Svar:
För att analysera detta räcker det med Newtons gravitationsteori, Einsteins gravitationsteori (allmänna relativitetsteorin) tillför inget betydelsefullt. Om en stjärna exploderar, förändras gravitationen inte alls för den utomstående betraktaren. Gravitationen från ett sfäriskt skal är nämligen precis densamma som om all materia var samlad i centrum. Det är först när den expanderande materien når betraktaren som gravitationen påverkas. Det sker inte på något drastiskt vis. Sitter man inne i ett sfäriskt skal, är gravitationen 0, alltså från det skal som är utanför. När gassfären expanderar, kommer en allt större del av den exploderade stjärnans materia att befinna sig utanför betraktaren. Gravitationen avtar succesivt. Alltså, i "normala" fall är förloppet så långsamt, att ljushastigheten inte spelar någon roll alls.

Sedan kan man ju tänka sig fall där det inte råder sfärisk symmetri. Där kommer då gravitationsstrålning att vara betydelsefull. Då måste man dra in den allmänna relativitetsteorin. Den är svårhanterlig matematiskt sett. Man jobbar mycket med dessa problem och man kan säga att det ännu är en bit kvar till lösningen.


/KS

*

Kraft-Rörelse [6917]

Fråga:
Hej! Hur skulle ni definiera massa? Det görs väl ganska mycket forskning på det här med massa? Vad har man kommit fram till? När förväntar fysikerna sig att kunna ge en fullständig förklaring till massans uppkomst?
/Rebecca A

Svar:
Massa är ett centralt begrepp i fysiken. Sedan gammalt skiljer man på den tunga massan (mg) och den tröga massan (mi), definierade genom

F = gmg

och

F = ami

där F är kraften, g är tyngdaccelerationen och a är accelerationen. Det är ingalunda givet att den tunga och tröga massan är lika. Redan Newton gjorde en del experiment, som visade att det i varje fall inte skiljde mycket. Ungraren Eötvö gjorde i slutet av 1800-talet en rad berömda försök, som visade att de var lika inom en miljarddel. Om du är intresserad av detta, finns det detaljerat beskrivet i Nationalencyklopedin . När Einstein 1915 formulerade den allmänna relativitetsteorin, postulerade han att den tunga och tröga massan är lika (ekvivalensprincipen). Det innebär att fysiken är lika för alla system i fritt fall (tyngdlöst tillstånd).

Experiment: Håll en sten i handen. Nu upplever du den tunga massan. Kasta iväg stenen. Nu upplever du i huvudsak den tröga massan.

En helt annan aspekt på massa är hur elementarpartiklarnas massa ska förklaras. Den mest populära förklaringen har med Higgs-partikeln att göra. Den är ännu inte upptäckt, men man hoppas göra det med den jätteaccelerator som kallas LHC, och är under byggnad vid CERN-laboratoriet utaför Geneve. Den bör komma igång år 2005.
/KS

Nyckelord: massa, trög/tung [4];

1 http://public.web.cern.ch/Public/
2 http://lhc-new-homepage.web.cern.ch/lhc-new-homepage/

*

Kraft-Rörelse [7676]

Fråga:
Hej! Det är en sak jag inte förstår med Einstein och hans relativitets teori: Ljusets hastighet är ju absolut, det är lika för alla oberoende om man är i rörelse eller ej, men då borde inte dopplereffekten gälla ljuset! Jag ska förklara lite hur jag tänker.

Jag har läst en bok som heter "Einstein för nybörjare" av Joseph Schwatz & Michael McGuinness. I den förklarar de väldigt bra hur det kan uppfattas att ljusets hastighet är absolut. Det beskrivs ungefär så här: Tänk dig ett tåg med en vagn i rörelse. I mitten av vagnen finns det en lampa och en man (Peter). Det finns en dörr i de båda ändarna av vagnen som vi låtsas kan öppnas av ljuset från lampan. Det finns även en man på marken utanför tåget (Anders). När Peter tänder lampan tycker han att dörrarna öppnas exakt samtidigt. Men Anders tycker att den bakre dörren öppnas aningen innan den främre dörren. Det gör han eftersom att ljuset från lampan rör sig med hastigheten c i förhållande till honom och den bakre dörren möter ljuset. Alltså öppnas den bakre dörren lite innan den främre.

Sen har jag också börjat läsa en bok som heter "Einsteins universum" av Nigel Calder. I den står det om dopplereffekten. När jag läser i den tycker jag inte att det stämmer med vad jag vet (tror att jag vet). Det står ungefär så här: "Om du sitter i en rymdraket och åker mot solen fortare och fortare kommer du tycka att solen blir blåare och blåare eftersom ljusets våglängder trycks ihop när du möter det. Kortare våglängd = energirikare ljus." Men ljusets hastighet beror ju på hur fort jag själv rör mig! Våglängderna borde inte kunna tryckas ihop. Jag förstår inte hur det fungerar!

Jag skulle också bli glad om ni kunde förklara för mig på något bra sätt varför längden på t.ex. den ovan nämda tågvagnen är relativ för Peter och för Anders.

Tack!
/Martin L, Långbergsskolan, Nyköping

Svar:
En orsak till att dessa saker är svåra att förstå, är att relativitetsteorin inte behövts i vardagslivet. Våra hjärnor har inte utvecklats så att vi ska ha det lätt med relativitetsteorin (eller kvantfysiken).

Kommentar till en av de sista meningarna: Ljushastigheten beror inte på hur fort du rör dig. Det är just utgångspunkten för relativitetsteorin. Det här var inget Einstein hittade på, det var experimentella resultat. Det stred mot Newtons mekanik, och Einstein insåg att den måste ersättas med något annat.

I stället för att kommentera de tankeexperiment du presenterar, vill vi hänvisa till en annan källa: Nationalencyklopedin . Artikeln där är mycket bra. Tyvärr har det insmugit sig ett par fel i figur 1, som gör den obegriplig. Observatören i rymdskeppet ska ha beteckningen B (har ingen beteckning nu). Den vänstra observatören på marken har beteckningen A (rätt), men den högra observatören har beteckningen B (fel, ska vara A). I ekvationen längst ner ska v ersättas med v2. Information om detta finns både i rättelseblad och supplementband. Tro nu inte att man kommer in i relativitetsteorin bara genom att läsa igenom artikeln. Man får nog ägna åtskilliga timmar åt det.
/KS

*

Kraft-Rörelse [7703]

Fråga:
Om vi antar att man på månen placerar en ljuskälla som pekar mot jorden, här på jorden så startas en raket mot månen i samma stund som ljuskällan utsänder en ljusimpuls. Vi antar at en betraktare står på jorden och en sitter i raketen, vem kommer ljuset att nå först(Om vi antar att de var på samma höjd vid startögonblicket)?
/Joakim M, Christofer Polhem, Visby

Svar:
Frågeställningen är korrekt formulerad för klassisk mekanik, men fel formulerad för relativitetsteorin. Vad menas med "först"? Einstein antog att ljushastigheten var konstant i alla system. Det var genom en noggrann analys av begreppet "samtidighet", som han kom fram till relativitetsteorin.
/KS

*

Ljud-Ljus-Vågor [7894]

Fråga:
I 1962 ble det gjennomført et eksperiment i forbindelse med Einsteins generelle relativitetsteori, der de hadde et vanntår og to klokker. Den ene klokka satte de nede ved bakken, den andre oppe i tårnet. Hvordan kan det ha seg at den oppe i tårnet gikk saktere enn den på bakken? Har det noe med lysets bølgelengde å gjøre, slik jeg en gang hørte det forklart; at når lyset går oppover mot atmosfæren, mister det energi, noe som gjør at frekvensen blir mindre og bølgelengden lengre og tiden går saktere?
/Vanja M, Horten vgs, Horten

Svar:
Klockan nere på marken går saktare än den uppe i tornet. Det har med gravitationen att göra. Vad är en klocka? Det är något som svänger, till exempel atomer. Man använder ju atomur i dessa sammanhang. Tittar man på de båda klockorna uppifrån, ser man att de skickar ut ljus med den karakteristiska våglängden. Den nedre klockans ljus måste ta sig upp ur en lägre gravitationspotential, och det kostar energi. Det enda sättet som ljuset kan "betala" för sig, är att våglängden ökar, det vill säga att frekvensen blir lägre. Klockan går alltså långsammare. Fenomenet kallas också gravitationell rödförskjutning.
/KS

Se även fråga 4276

*

Ljud-Ljus-Vågor [8428]

Fråga:
Relativitetsteorin

Vad jag har hört så fotograferades stjärnhimmlen under en solförmörkelse år 1919. Dessa foton jämfördes sedan ett år senare med nya foton av samma stjärnhimmel. Stjärnorna hade flyttat presic så mycket på sig som Einstein förutsagt, detta bekräftade relativitets teorin. Vad var poängen med att fota himmlen.

mvh Andreas
/Andreas O, Uppsala

Svar:
Poängen är just det du berättar om, nämligen att mäta stjärnornas positioner. Solens gravitationsfält böjer av stjärnljuset.
I själva verket var mätningarna från 1919 ganska dåliga, mätfelen var stora. Numera har vi 1000 gånger bättre mätnigar med två olika metoder:

1. Positionsbestämningar av stjärnor med Hipparcossatelliten.

2. Positionsbestämningar av kvasarer med sammankopplade radioteleskop på båda sidor av Atlanten. Kvasarer är mycket avlägsna massiva svarta hål, som suger upp materia och skickar ut enorma mängder strålning.

Båda mätningarna stämmer med den allmänna relativitetsteorin inom mätfelen.
/KS

*

Kraft-Rörelse [9014]

Fråga:
Jag har på sistonde blivit väldigt intresserad av Einsteins teorier. Nu har jag läst hans teori om hur tiden förändras beroende på hastighet. Jag tror mig förstå vad han menar, men jag håller inte över huvud taget med! Om jag reser till en galax 25 ljusår härifrån, och min kompis stannar på jorden. Jag sitter i rymdfärgan och åker i ljusets hastighet till denna galax... det tar mig 25 år. Jag sitter i rymdfärgan i 25 år, och blir 25 år äldre. På jorden, under samma tid, väntar min kompis på att jag ska nå galaxen... han väntar i 25 år, och blir 25 år äldre. Vi har nu båda blivit 25 år äldre?! Om det tar ljuset 25 år att komam dit, och jag reser i ljusets hastighet, så tar det väl 25 år och inget annat!
/Paolo V, Filmgymnasiet, Malmö

Svar:
Du kan inte färdas med ljushastigheten, det skulle kräva oändlig energi. Sedan resonerar helt med klassisk fysik, inte med relativitetsteori.

En klocka som färdas fort går långsammare. Det är inte abstrakt teori utan ett vardagligt fenomen för många människor, till exempel partikelfysiker. Vi kan nämna ett fall som direkt berör dig. Högt uppe i atmosfären (c:a 25 km) producerar kosmiska strålningen ett slags partiklar som kallas myoner. De har en medellivslängd på 0.0000022 s. De färdas med nära ljushastigheten (300000 km/s). Multiplicerar vi dessa tal får vi 0.66 km. Så långt skulle alltså myonera gå enligt klassisk fysik. Ändå passerar dussinvis av dessa partiklar genom din kropp varje sekund. Det beror på att deras klocka går långsammare, i typiska fall 100 - 1000 gånger långsammare.
/KS

Se även fråga 7804 och fråga 7398

*

Blandat [9021]

Fråga:
Jag har hört ett exempel på relativitetsteorin som gick ut på att en man reste iväg från jorden med en snabb rymdfarkost, och när han kom tillbaka hade han åldrats mindre än de som stannat kvar på jorden. Min fråga lyder: Mot vilken referenspunkt mäter man hastigheten? Man kan väl lika väl säga att jorden rör sig snabbt i förhållande till rymdskeppet?
/david ö, visby

Svar:
Detta är den så kallade tvillingparadoxen, som inte alls är någon paradox. Hastigheten är inte det viktiga utan accelerationen. Det ena systemet är accelererat, det andra inte. "Paradoxen" uppkommer genom att Einstein inte var någon radikal matematiker när han 1905 formulerade den speciella relativitetsteori. Bara 3 år senare formulerade Minkowski om den med en ny metrik. I fortsättningen accepterade Einstein helt Minkowskimetriken, där tvillingparadoxen knappast kan formuleras. Det är därför konstigt man fortfarande, efter nästan 100 år, lär ut relativitetsteorin med Einsteins klassiska metrik. Kolla svaren och länken nedan!
/KS

Se även fråga 8426 och fråga 4828

Nyckelord: tvillingparadoxen [5];

1 http://mentock.home.mindspring.com/twins.htm

*

Kraft-Rörelse [9182]

Fråga:
Hej Jag har tänkt skriva ett arbete om teleportering och kvantkorrelatering (ickelokalitet). Av att dömma av de fakta jag hittat på internet då bygger desssa fenomen på en hel del avancerad matematik. Så min fråga till er är om ni har tips på någon bok eller internetsite där detta förklaras på ett lättare sätt, helst på svenska. (jag går NV3e)

sen måste jag fråga: Einsteins idée om att de inte finns nån gravitiation utan att massor påverkar varandra pga deras böjning i rummet. Är denna teori omkullkastad av standard modellen med en tänkt gravitaionsförmedlar-partikel?

Tack på förhand
/Erik N, Östra-Gym, Umeå

Svar:
Det finns en bra artikel om teleportering i Forskning och Framsteg, 2001, nr 2 sid 44, Forskning & Framsteg, webbarkivet . Den är skriven av den österikiske fysikern Anton Zeilinger som verkligen är expert på området. Den kräver inga matematiska förkunskaper. Tro inte att du kommer begripa det hela, det går inte. Som du riktigt påpekar, är den yttersta förklaringen att vårt universum är ickelokalt, varje del av universum står i momentan kontakt med varje annan del. Redan Newton funderade över dessa saker. Det finns absolut acceleration men inte absolut rörelse. Varför?

Sen till din andra fråga. En fysikalisk teori gör inte anspråk på att ge den sanna beskrivningen av verkligheten. Vi ska snarare se den som en modell, som är mer eller mindre användbar i olika sammanhang. Den allmänna relativitetsteorin (som ju är en gravitationsteori) är en teori som inte innefattar kvantmekaniken. Det är till och med så illa att de båda teorierna är motsägelsefulla. Det betyder inte att Einsteins gravitationsteori är fel. Alla experimentella tester stämmer överens med den, i motsats till konkurerande gravitationsteorier.
/KS

Se även fråga 4921 och fråga 1820

*

Kraft-Rörelse [9324]

Fråga:
Hej! Undrar lite om skillnaderna mellan den klassiska fysiken och Einsteins två relativitetsteorier. T.ex. innan dessa var det allmänt accepterat att och rum var absolut, men dessa postulat motbevisades av Einstein och förändrade vårt sätt att betrakta universum!

Nu undrar jag vilka andra fenomen som den klassiska fysiken inte kunde förklara, fick sina förklaringar genom Einsteins eminenta teorier? Vore tacksam för svar.
/Magnus A, Köping

Svar:
Valet av ordet "relativitetsteori" är lite olyckligt. Ofta får man höra att den innebär att "allt är relativt", vilket är en missuppfattning. Både Newtons och Einsteins teori (från 1905) är relativitetsteorier i den mening, att relativitetsprincipen gäller i båda. Denna säger att fysiken är densamma i inertialsystem, alltså system i likformig rörelse.

Den viktiga skillnaden är, att Einstein utgår ifrån, att ljushastigheten i vakuum är konstant och oberoende av inertialsystemet. Vidare utgår han ifrån att energi och information inte kan förflytta sig snabbare än ljuset. Det leder till att begrepp som tid, längd och samtidighet inte fungerar enligt vår vardagsuppfattning. Det leder också till den välkända relationen mellan energi och massa E = mc2.

Den allmänna relativitetsteorin är huvudsakligen en gravitationsteori, baserad på den speciella relativitesteorin. Här inför man ekvivalensprincipen, som säger att den tunga massan är lika med den tröga massan. Den första har med gravitation att göra, den andra med acceleration. Gravitationskraften (liksom centrifugalkraften) betraktas här som en fiktiv kraft, alltså en kraft som egentligen inte behövs. Partiklar som inte påverkas av någon kraft, rör sig "rätlinjigt" i den krökta fyrdimensionella rumstiden. En satellit som rör sig i en bana runt jorden, rör sig i någon mening "rätlinjigt" i den rumstid, som kröks av jordens massa.

Att gå in på alla fenomen, som förklaras av Einsteins teorier skulle dra alldeles för långt. Man kan i alla fall konstatera, att inga exerimentella data strider mot teorierna. Sedan vill vi påpeka, att man inte kan säga att Newtons mekanik är fel. Under mindre extrema förhållanden duger den utmärkt. När man skickar sonder till Mars, använder man Newtons mekanik.
/KS

Nyckelord: massa, trög/tung [4]; relativitetsteorin, allmänna [33]; relativitetsteorin, speciella [45];

*

Kraft-Rörelse [9519]

Fråga:
Det er alltså mulig å velge et referansesystsem for å beskrive et legemes tilstand, som for eksempel når jeg vil beskrive fart osv til et tog, velger jeg selvsagt bakken som referansesystem. Hvis jeg nå tar en lommelykt og sender lys foran meg, kan jeg da selv velge referansesystem? Kan jeg si: lyset står stille, jeg flyr bakover? Det kan umulig stemme, men hvorfor stemmer det ikke? Takker for svar
/Vanja M, Horten vgs, Horten

Svar:
Det viktiga med den speciella relativitetsteorin är inte att den är en relativitetsteori. Också Newtons mekanik är en relativitetsteori, det påpekar Einstein själv. Nej, det viktiga är att ljushastigheten är konstant i alla inertialsystem (system med likformig rörelse).

Om du tänker igenom detta, kan du säkert själv besvara din fråga.
/KS

*

Kraft-Rörelse [9833]

Fråga:
vad är massa
/viktor j, prästängen, eksjö

Svar:
Sedan gammalt skiljer man på två olika massbegrepp, den tunga massan och den tröga massan. Redan Newton funderade på dessa saker. I den allmänna relativitetsteorin postulerar Einstein att de är lika. Kolla svaren nedan!
/KS

Se även fråga 6917 och fråga 7923

*

Kraft-Rörelse [9857]

Fråga:
Hej! Jag håller på med ett projekt som handlar om relativitetsteorierna och jag har kört fast. Jag undrar om varför man, trots att Newtons teorier om mekanik och gravitation, fortfarande använder sig av dennes teorier när Einstiens teori är mer precis? Är också intresserad av de filosofiska konsekvenserna av den speciella relativitetsteorins postulat? Kort sagt jag vill gärna veta varför man väljer Newtons formler istället för Einsteins? Är Einsteins för komplicerade eller hur ligger det egentligen till? Vore mycket tacksam för svar! Tack på förhand, m.v.h. Magnus
/Magnus A, Ullvi gymnasium, Köping

Svar:
Newtons mekanik är en bra approximation av den speciella relativitetteorin vid måttliga hastigheter. Den räcker helt för NASA´s interplantära rymdsonder.

Det centrala posulatet i den speciella relativitetsteorin är att ljusets hastighet i vakuum alltid är konstant, oberoende hur vi rör oss. Det strider ju mot vår vardagserfarenhet. Här kan man addera hastigheter, det kan man inte i relativitetsteorin.
/KS

*

Kraft-Rörelse [9884]

Fråga:
Einstein sier selv at relativitetsteorien sier noe om hvordan ur og staver oppfører seg fysisk. Dermed, når en snakker om masseøkning i denne forstand, må det være slik at massens partikler "blåser seg opp" på en eller annen måte. Men hvis også dette skal gjelde begge veier, sett fra system A blåser B seg opp, og omvendt, er det snakk om et synsbedrag, og dermed ligger det ikke noe fysisk i det. Jeg har kommet fram til at den spesielle relativitetsteorien er et matematisk redskap som ikke forteller om noe fundamentalt ved naturen, som antakelig den generelle gjør. Riktig eller galt? Takknemlig for svar.
/vanja M, Horten vgs, Horten

Svar:
Visst är massökningen ett reellt fysiskt fenomen. Vid de avslutade LEP experimenten vid CERN var elektronerna och positronerna 200000 gånger tyngre än stillastående partiklar. Högst påtagligt. Detta beror ytterst på att ljushastigheten är konstant i alla inertialsystem, något som strider mot sunt förnuft. Då kan man väl säga att speciella relativitetsteorin verkligen säger något fundamentalt om naturen!

Den inkluderar inte gravitationen. Den allmänna relativitetsteorin är en relativistisk gravitationsteori.
/KS

Se även fråga 9883

*

Kraft-Rörelse [9980]

Fråga:
Vad har man för andvändning för Einsteins relativitetsteorin?
/Anna L, Stagnelius, Kalmar

Svar:
Speciella relativitetsteorin är bra vid höga hastigheter och stora avstånd. Är där dessutom starka gravitationsfält, behöver man den allmänna relativitetsteorin.
/KS

Se även fråga 9958

*

Kraft-Rörelse [10317]

Fråga:
Varför är tiden en dimension? Är det bara något man säger eller finns det ekvationer bakom påståendet?
/Robert W, Lund

Svar:

I Einsteins ursprungliga formulering av relativitetsteorin år 1905 behandlas tiden inte som en dimension. Några år senare lade
H Minkowski fram en ny version, där tiden behandlades som en fjärde dimension, den så kallade Minkowskimetriken. Den visade sig ha stora fördelar. Tvillingparadoxen går inte ens att formulera.
Einstein accepterade helt den nya metriken, som han senare använde i den allmänna relativitetsteorin.
/KS

Se även fråga 9958 och fråga 4828

*

Ljud-Ljus-Vågor [10843]

Fråga:
Ett av de mest kända bevisen för att Einsteins relativitetsteori är ett användbart sätt att beskriva naturen är en ljusstråles avböjning när den passerar ett mycket tungt föremål , t. ex. en planet. Nu kommer min fråga. Böjs en ljusstråle på samma sätt när den passerar genom ett mycket starkt elektriskt eller magnetiskt fält?
/Mats C, Hjalmar Strömerskolan, Strömsund

Svar:
Gravitationen finns inte med i den speciella relativitetsteorin, så detta är en effekt i den allmänna relativitetsteorin. Där behandlas gravitationen som en krökning av rummet. Elektriska eller magnetiska fält har ingen sådan inverkan på rummet, utom möjligen i mycket extrema situationer.
/KS

*

Elektricitet-Magnetism [10904]

Fråga:
Hej! Man hör ofta om James Clerk Maxwells - jag tror det var fyra - ekvationer för elektromagnetismen. Jag kan trots detta inte erinra mig att jag sett alla, eller ens någon av dessa. Hur ser de ut?
/Jimmy K, Martin Koch-gymnasiet, Hedemora

Svar:
På sajten nedan hittar du en variant av Maxwells ekvationer. De är en elegant sammanfattning av vad man visste om elektromagnetism på 1800-talet. Tidigt insåg man att där fanns en hake. De uppfyller inte relativitetsprincipen, alltså att fysikens lagar är samma i olika inertialsystem (system i likformig rörelse). Många försök gjordes att modifiera ekvationerna, men utan framgång. År 1905 löstes problemet av Albert Einstein genom den speciella relativitetsteorin.

Se Maxwells ekvationer .
/KS

Se även fråga 3566

*

Ljud-Ljus-Vågor [11283]

Fråga:
Hej! Har länge funderat över hur ljuset färdas genom vakuum dvs,hur transporteras det vilket färdmedel använder det sig av.Det verkar som om Einstein kringgår problemet.En foton är väl inte självdrivande med inbyggda turbos?Min logik säger mig att det måste finnas någon form av eter eller medium för att ljuset ska färdas i vakuum eller färdas överhuvudtaget? Utesluter inte Einsteins relativitetsteori just ett medium eller etern som transportmedel?Jag hoppas att ni förstår min fråga,och vore sedan tacksam för ett svar!
/Bengt O, Åsö, Sthm

Svar:
Vakuum är inte utan egenskaper. Det har faktiskt både elektriska och magnetiska egenskaper, som bestämmer ljushastigheten, men vi talar inte om "eter" nuförtiden. Kolla svaret nedan!
/KS

Se även fråga 9897

*

Ljud-Ljus-Vågor [11534]

Fråga:
Hej! Jag undrar hur eller om satelliter utnyttjar einsteins relativitetsteori, och om alla satelliter måste göra detta. Dessutom undrar jag om ni kan hjälpa mig svara på hur einstein direkt eller indirekt påverkat skapandet av solceller och lasrar.
/majd s, polhem, lund

Svar:
För den första frågan hänvisar vi till svaret nedan. Einstein gav den teoretiska förklaringen till den fotoelektriska effekten. Det var för det han fick nobelpriset, se The Nobel Prize in Physics - Laureates . Han förutsade teoretiskt stimulerad emission, som är det fenomen lasern bygger på.
/KS/lpe

Se även fråga 11535

*

Kraft-Rörelse [11546]

Fråga:
Hej igen! Jag uttryckte mig kanske lite luddigt. Vad jag menar är alltså följande:

Finns det någon sorts tröghet i ett föremål som är viktlöst, eller uppför sig ett föremål likvärdigt oavsett massa när den nu är viktlöst?

Alltså....ponera att jag finns i ett rymdskepp där viktlöshet rådet. Jag är fastspänd mot väggen. "Olle" slänger ett föremål mot mig som på jorden skulle ha vägt 1000kg. Nu gör han samma sak, men denna gång med ett föremål som på jorden endast skulle ha vägt 1kg. Hastigheten är densamma i båda fallen. Min fråga är alltså....skulle "skadorna" bli dom samma av båda föremålen eftersom dom är viktlösa, eller rådet det nån sorts tröghet som gör att jag skulle bli betydligt mer "tillplattad" om jag träffades ev föremålet som väger ett ton? Man skulle också kunna ställa frågan så här. Krävs det mer energi från min sida att kasta iväg ett föremål med en STOR massa i 10km/h, än ett föremål med liten massa? Båda är ju viktlösa. Om svaret är ja, förklara då varför det är så eftersom allt är viklöst i rymden.

Mvh Calle
/Calle R, Tallbacka, Värnamo

Svar:
Vi envisas: Använd inte ordet "viktlös", använd i stället "tyngdlös", för det är vad det handlar om. "Tillplattningseffekten" beror på rörelseenergin

E = mv2/2

och massan är ju alltid densamma.

Är det självklart? Är den massan som gör motstånd mot acceleration (tröga massan) densamma som tyngdkraften känner av (tunga massan). Redan Newton insåg att detta inte behöver vara fallet. Han gjorde en del experiment som i alla fall visade att de var ungefär lika. I sin allmänna relativitetsteori utgår Einstein att de är lika (Ekvivalensprincipen).
/KS

Se även fråga 11533 och fråga 6917

*

Kraft-Rörelse [11658]

Fråga:
Hur härleder man fullständigt matematisk (med relativitetsteori) uttrycket för händelsehorisonten för svarta hål? R = 2GM/c^2
/Bo J, Gripenskolan, Nyköping

Svar:
Att presentera en sådan härledning här skulle vara över denna frågelådas ambitionsnivå. Vi får hänvisa till en någorlunda modern lärobok i allmän relativitetsteori.

Schwarzschilds exakta lösning (år 1916) av Einsteins ekvationer gäller ett sfäriskt symmetriskt system. Den generella lösningen för ett roterande system gjordes av Roy Kerr år 1963. Den tas inte upp i detalj i elementära läroböcker. Här är det inte så enkelt att man bara har en väldefinierad händelsehorisont.
/KS

*

Universum-Solen-Planeterna [12745]

Fråga:
Eftersom alla forskare numer är helt ense om att svarta hål existerar har jag en fråga rörande detta ämne.

Skulle det kunna ske och vad skulle i så fall hända om två svarta hål kom tillräckligt nära varandra för att det tyngre av dem skulle kunna suga ner det andra i sig självt?
/Thomas L, Centralskolan, Laxå

Svar:
Ett svart hål är en koncentration av massa med ett så starkt gravitationsfält att ingenting, inte ens ljuset, kan övervinna kroppens gravitation. Materia eller ljus som kommer in innanför det svarta hålets händelsehorisont förblir där och kan aldrig komma ut igen, förutom eventuellt oerhört långsamt i form av Hawkingstrålning (se fråga 6026 ).

Händelsehorisonten (Schwartzschild-radien) ges av

rS = 2GM/c2

där G är gravitationskonstanten, M är hålets massa och c är ljushastighen.

Det är helt riktigt att man numera är helt övertygad om att "svarta hål" existerar.

Om två kroppar (stjärnor, svarta hål, eller egentligen vilka som helst bara de har massa) kommer tillräckligt nära varandra för att gravitationen mellan dem inte är försumbar kommer de att börja röra sig i längs en bana kring sin gemensamma tyngdpunkt.

Einsteins allmänna relativitetsteori säger att om kropparna har mycket stor massa (som t.ex. svarta hål) kommer deras rotation att påverka rumstiden i deras närhet - den börjar "svänga", och vad vi kallar gravitationsvågor bildas. Gravitationsvågorna leder bort energi från systemet, vilket leder till att de svarta hålen kommer närmare och närmare varandra för varje varv de snurrar. Till slut möts de.

Exakt vad som händer när de två hålen möts är mycket svårt att säga - experterna kör komplicerade och tidskrävande datorsimuleringar för att ta reda på detta. Alla beräkningar är dock ense om att det dels bildas ett "nytt" och massivare svart hål, dels sänds en extremt kraftig gravitationsvåg ut.

Just nu jobbar flera forskargrupper runt om i världen på att konstruera jättelika detektorer för att kunna detektera gravitationsvågor. Ett sådant instrument kallas LIGO, Laser Interferometer Gravitational Wave Observatory - läs mer om detta under länk 1.

Även med "vanlig" astronomi har man gjort observationer av vad som med hög sannolikhet är följden av en kollision mellan två svarta hål, se bilden nedan (från länk 2) som är tagen med ett radioteleskop. Den lilla bilden i cirkeln visar galaxen NGC 326 som den såg ut innan hålen kolliderade - det svarta hål som befann sig i mitten av galaxen slungade ut strålning i två smala koner ("propellerbladen"). En senare mätning (stora bilden) visar att något drastiskt har skett - strålningen skickas nu ut i helt andra riktningar, eftersom det svarta hålets massa och riktningen på dess rotationsaxel har kraftigt ändrats.

Se vidare den mycket bra Wikipedia-artikeln Svart hål. Se även No-hair_theorem och Charged_black_hole .

Försök att fotografera ett svart hål:
http://www.space.com/36360-black-hole-image-event-horizon-telescope.html

Se även fråga 18930 .



/Margareta H/lpe

Nyckelord: svart hål [51]; gravitationsvågor [19];

1 http://www.ligo.caltech.edu/
2 http://spaceflightnow.com/news/n0208/02blackholes/

*

Kraft-Rörelse [12753]

Fråga:
Hur kom Einstein på formeln E=mc2?
/Veckans fråga

Ursprunglig fråga:
Jag har en fråga om formeln E=mc2. Jag undrar lite över hur folk (fysiker) reagerade när formeln publicerades. Var man skeptiska eller lyriska? Trodde man på att den verkligen fungerade från början?

Hur kom Einstein på formeln E=mc2? Var det en slump, eller inte? (Henrik A)

Fungerar alltid Einsteins formel E=mc2? Vad använder man den till? (Erik D)
/Niklas N, Äppelviken, Bromma

Svar:
Jag har tagit mig friheten att slå ihop frågorna från Äppelviksskolan.

Det är svåra frågor ni ställer eftersom ni vill veta det historiska perspektivet. Det är en hel vetenskap som heter vetenskapshistoria, och det är vi inte experter på. Jag skall emellertid försöka mig på ett par kommentarer.

Relativitetsteorin publicerades av Einstein 1905. Här är originalversionen: Zur Elektrodynamik bewegter Körper . Artikeln är svårbegriplig för en modern fysiker, eftersom beteckningarna i formlerna är lite gammalmodiga och så är artikeln på tyska. Vid den tiden var emellertid tyska mer vetenskapens språk än engelska, och tidens fysiker hade inga större problem att förstå artikeln.

Relativitetsteorin bygger i stort på ett antagande: att ljushastigheten c i vakuum är konstant oberoende av hur man rör sig i förhållande till ljusstrålen. Antagandet bygger på ett experiment som utfördes av Michelson-Morley 1887: History of Special Relativity . Ovanstående artikel är mycket bra, och bör ha övertygat många fysiker. Artikeln innehåller nästan hela var vi kallar den Speciella relativitetsteorin utom det ni frågar om, E=mc2. Denna härleddes i en artikel publicerad senare under 1905: Does the Inertia of a Body Depend upon Its Energy-Content . Även denna artikel är svårläst i dag (trots att den här är översatt till engelska). I artikeln finns en länk till en engelsk översättning av Einsteins första artikel om relativitetsteorin (Zur Elektrodynamik bewegter Körper).

Länk 1 nedan är Einstein själv som förklarar vad formeln innebär. Här är en artikel som ger ett par olika härledningar: Year of Physics 2005 .

Exakta mätningar som bekräftade relativitetsteorin kom betydligt senare. Inte ens nobelkommittén var imponerad: nobelpriset Einstein fick 1921 var för hans förklaring av den fotoelektriska effekten (också 1905) och inte relativitetsteorin!

Tolkning: Ekvationen E=mc2 skall tolkas så att massa och energi är ekvivalenta (utbytbara mot varandra) med "växlingskursen" c2. Eftersom c är ganska stort motsvarar även en liten massa (eller mass-skillnad) en mycket stor energi.

Bekräftelsen på E=mc2 kom först på 20/30-talet när man kunde mäta atommassor med hög precision, se fråga 12726 .

Det finns många böcker om relativitetsteorin och en bra artikel i Nationalencyklopedin . Wikipedia-artiklarna Einstein och Special_relativity är också mycket bra.

Se även fråga 20460 .



/Peter E

Nyckelord: Einstein, Albert [1]; relativitetsteorin, speciella [45];

1 http://www.aip.org/history/einstein/voice1.htm

*

Kraft-Rörelse [16683]

Fråga:
Hej Kan ni förklara ingående för mig varför Merkurius bana har vissa avvikelser. Jag läste att Einsteins gravitationsteori gav svar på detta men det stod inte varför. Tacksam för svar.
/Anna S

Svar:
Merkurius periheliumprecession: Perihelium för en elliptisk bana är den punkt som är närmast centralkroppen. Enligt klassisk mekanik ligger denna bana fixt i rymden. Enligt den allmänna relativitetsteorin roterar perihelium sakta kring centralkroppen, se nedanstående figur från Wikimedia Commons.

För Merkurius hade man redan före relativitetsteorin mätt upp denna precession till 43" per århundrade, men man hade fått ta till en okänd planet - Vulcan - för att förklara den.

Einsteins förklaring med den allmänna relativitetsteorin var den första experimentella bekräftelsen på teorin.

En härledning av uttrycket för precessionen finns i Wikipedia-artikeln Kepler_problem_in_general_relativity , med den innehåller ganska avancerad matematik!



/Peter E

Nyckelord: relativitetsteorin, allmänna [33];

*

Kraft-Rörelse [11977]

Fråga:
Hej!

Jag undrar om man kan härleda Einsteins formel för rörelseenergi? Den som gäller vid höga hastigheter nära ljusets. Kan man härleda denna och E=mc^2 rent matematiskt?

Vad jag förstår så kan man sätta att

E(rörelseenergin)=mc^2/roten ur 1-v^2/c^2 - mc^2 är ungefär lika med mv^2/2?

Hur härleder man i så fall det?

Försöker inte bara teoretiskt utan också rent matematiskt förstå denna formel!

All hjälp emottas tacksamt!! Laila

E-mail: lailacarina@hotmail.com
/Laila N, Rudbeck komvux (Lernia), Sollentuna

Svar:
Jovisst kan man härleda E = mc2, men det är en uppgift för en lärobok i relativitetsteori. Detta följer av de två postulat Einstein utgår ifrån:

1. Fysikens lagar är samma i alla system i likformig rörelse (relativitetsprincipen).

2. Ljushastigheten i vakuum är alltid densamma.

Den senare är det svårt att få grepp om. Den strider mot vardaglig erfarenhet av föremål i rörelse. Den leder till att den transformation man använder i klassisk mekanik (Galileitransformationen) måste ersättas av Lorentztransformationen. Där spelar Lorentzfaktorn

g = 1 / (1 - v2/c2 )½

stor roll.

Svar på den senare frågan: Det är bara att räkna på. Man får göra en del aproximationer. Den viktigaste är:

( 1 - a )½ = 1 - a/2 (nästan)

för a << 1.
/KS

*

Blandat [12106]

Fråga:
Man har konstaterat att ljusets hastighet i vakuum är konstant 299792458 m/s (s=tidsenhet). Men enligt Einsteins relativitets-teori kan tiden variera beroende på rörelse. Borde inte ljusets hastighet vara omöjlig att bestämma eftersom tiden inte är konstant?
/Maria H, Upplands-Bro Gymnasium, Kungsängen

Svar:
Det är just det faktum att ljusfarten är konstant som gör att tiden blir relativ.

Eftersom hastighet = (sträcka)/(tid) så ändrar sig både sträcka och tid med rörelsen men på ett sådant sätt att ljusfarten blir konstant.
/Gunnar O

*

Energi [12172]

Fråga:
Jag undrar hur nutiden har påverkats av Einsteins relativitetsteori allra tydligast? Han har ju förädrat oändligt mycket inom naturvetensksap, men vad äknas som huvudsaken, om det finns en sådan?
/johanna l, wennöska, malmö

Svar:
Jag antar att du avser den speciella relativitetsteorin. De flesta effekter i denna märks bara på föremål som rör sig väldigt fort - nära ljusets hasighet 300.000 km/sekund. Därför är säkert den del av relativitetsteorin som betytt mest formeln E=mc2 - ekvivalensen mellan massa och energi. Denna innebar att man kunde "förstå" hur solen producerar sin energi och hur man får energi från kärnkraftverk.
/Peter E

Se även fråga 1459 och fråga 8091

*

Kraft-Rörelse [12403]

Fråga:
Min fråga är angående gravitationen. Jag kan inte riktigt beskriva det själv, så jag får ta det genom exempel: Jordens gravitation, kan enligt mig, liknas vid en tvättmaskin när den centrifugerar, dvs allt dras åt trummans väggar, bara det att man har vrängt trumman ut och in fast att den fortfarande har samma egenkaper, dvs att allt dras åt trummans väggar.

Om man då tar jorden som exempel, då säger jag mig att allt på jordens yta borde slungas ut i rymden, men att det hindras av gravitationen. Men vad är då gravitationen, dvs det som gör att allt stannar kvar på jorden när den snurra. Jag jämför jorden med en slags roterande cylinder, där man har satt fast te x några sandkorn, och när den börjar rotera snabbt, skulle sandkornen slitas loss.

Eller kort och enkelt: Vad är det som gör att gravitationen verkar på jorden som den gör (det som gör att saker dras mot jorden i stället)?
/Anton L, Västerskolan, Uddevalla

Svar:
Det är korrekt att verkan av gravitation och acceleration är besläktade, eller t.o.m. ekvivalenta enligt Einsteins allmänna relativitetsteori. Om man tvingar något (t.ex. kläderna i den centrifugerande tvättmaskinen) att gå i en cirkel, så utsätts det för en acceleration in mot centrum, vilket enligt ekvivalensprincipen kan ses som en "gravitation" ut från centrum.

För ett löst föremål på den roterande jorden har du två effekter: en utåtriktad kraft p.g.a. jordens rotation och en inåtriktad kraft p.g.a. gravitationkraften från jordens massa. Turligt nog är den första effekten mycket mindre än den andra. Om jorden haft mycket mindre massa eller roterat mycket fortare, hade inte bara sandkorn på ytan flugit iväg, utan hela jorden hade slitits i stycken!
/Peter E

*

Kraft-Rörelse [12597]

Fråga:
Jag undrar lite över relativistiska effekter. Ser observatörer utanför ett föremål som rör sig (1-10^(-99))c att det rör sig i denna hastighet? Om det tar en sekund enligt föremålet att röra sig en viss sträcka, bör det betyda att en observatör tycker att det tar oändligt lång tid för föremålet att röra sig samma sträcka. Eller? Det i sin tur bör betyda att föremålet rör sig mycket långsammare enligt observatören.

Jag har läst att Einstein ansåg att all materia rör sig i ljusets hastighet uppdelat i tre rumsdimensioner och en tidsdimension. Ju snabbare genom rummet man rör sig ju långsammare genom tiden. Eftersom inget förutom ljuset kan röra sig i ljusets hastighet bör det betyda att att rörelse genom rummet ej kan ske om inte den kompletteras med en rörelse genom tiden. Står tiden stilla kan inte en rörelse genom rummet noteras. Men varför gäller detta inte ljuset? Varför ser vi att ljus rör sig trots att tiden står stilla för ljuset enligt oss? Vad är det jag har missat?
/Adel A, Christopher Polhem, Visby

Svar:
Så här skall du föreställa dig detta utan att få motsägelser:

Den rörliga observatören (egentligen kan man inte skilja på observatörerna, men antag att den ena observatören är på jorden och den andra i en raket) ser den tillryggalagda stäckan mycket förkortad - lorentzkontraktion.

Den stationära observatören tycker att tiden i det rörliga systemet går mycket långsammare - tidsdilatation.

Resultatet blir då att de ändå är överens systemens relativa hastighet.

Rörelse är ju tillryggalagd sträcka på en viss tid, så rörelse utan tid är meningslös.

Apropå speciella relativitetsteorin, här är en klassisk limerick:

  There was a young Lady named Bright,
  Whose speed was far faster than light.
  She went out one day
  in a relative way
  And returned on the previous night!

- A. H. Reginald Buller (1874-1944)
/Peter E

Se även fråga 10432 och fråga 2697

Nyckelord: relativitetsteorin, speciella [45];

*

Universum-Solen-Planeterna [12602]

Fråga:
När började den moderna kosmologin?
/Veckans fråga

Ursprunglig fråga:
Hej, jag har ett specialarbete om moderna kosmologin, när började den?
/Mohammad H, Brännkyrkagynasium, Midsommarkransen

Svar:
Hej Mohammad! Det är omöjligt att svara på eftersom begreppet 'modern kosmologi' inte är väldefinierat. Man kan säga på 1600-talet då en korrekt världsbild började växa fram. Fler milstolpar: Einsteins allmänna relativitetsteori 1916; upptäckten av Hubble att universum expanderar (1929); Gamows formulering av Big Bang teorin (40-talet); förståelsen i början av 50-talet hur sjärnorna producerade sin energi och hur grundämnena bildats; Penzias och Wilsons upptäckt 1965 av den kosmiska mikrovågsstrålningen; HST (HubbleSite ); de senaste resultaten från Wilkinson Microwave Anisotropy Probe (WMAP) som ger mycket noggrann information om universum, bland annat vad universum består av, se bilden nedan från "NASA/WMAP Science Team".

Börja med att slå på kosmologi i Nationalencyklopedin , där finns en bra artikel av Peter Nilsson. Wikipedia har ett par bra artiklar som mycket detaljerat behandlar även historiska aspekter: Cosmology och Timeline_of_cosmology . Sedan finns det många källor att fortsätta med, ovanstående och länken nedan ger en början.



/Peter E

Nyckelord: kosmologi [33];

*

Blandat [12616]

Fråga:
Hej! Jag undrar var jag kan hitta bra material angående relativitets teorin. Jag som frågar är lärare och vill hitta bra material till en elev som skriver ett special arbete om just relativitets teorin.
/Karolina S

Svar:
Hej Karolina! En bra startpunkt för den som är nyfiken på relativitetsteorin, men inte har hunnit läsa så mycket matematik ännu, är att söka information om Albert Einstein. En bra kort sammanfattning av hans liv och verksamhet (på svenska) finns på Fysikgöken - Fysikhistoria (klicka på Fysikerna och välj Einstein i den alfabetiska listan).

En trevlig site (på engelska) med intressanta animationer som illustrerar några av relativitetens grundläggande begreppen är Fear of Physics (länken heter Einstein's Relativity).

Mer detaljerad information hittar man sen t.ex. i Nationalencyklopedin (slå på relativitetsteori eller på Einstein).
/Margareta H

Se även fråga 7676 och fråga 10816

*

Kraft-Rörelse [12833]

Fråga:
I vår kursbok står det "Alla föremål faller lika fort om de inte påverkas av något luftmotstånd". Samtidigt vet man ju att tyngdkraften som ju då är den enda återstående(?) kraften beror på massorna hos de två föremålen. Vilket stämmer då? Faller en kula som har massan 2 ton fortare eller lika fort som en lika stor kula med massan 2 g? VARFÖR!!?
/tero l

Svar:
Alla massor faller lika fort i ett tyngdkraftfält. Det är korrekt att tyngdkraften på en större massa är större än den på en mindre massa i samma tyngdkraftfält, men MOTSTÅNDET MOT ACCELERATION (tröghet) är större i exakt samma proportion. Detta till synes enastående tillfällighet gav upphov till mycket diskussion fram till Einsteins allmänna relativitetsteori, se vidare nedanstående svar.
/Peter E

Se även fråga 12834

*

Kraft-Rörelse [12834]

Fråga:
Är Newton alltid konstant?

Om det är det, hur kan då Newton ha ett konstant förhållande gentemot en massas vikt i kg (c:a 9,8), då en massas vikt i kg varierar beroende på hur stark gravitationskraft massan utsätts för? T.ex. väger en människa olika mycket i kg beroende på vilken planet hon befinner sig på, hur kan hon då alltid ha ett konstant Newton-värde?
/Pontus X, Skärholmen, Stockholm

Svar:
Pontus! Om jag förstår dig rätt så undrar du varför det man kallar tröga massan m1 (mängd materia) är lika med den tunga massan m2 (hur massan påverkas av ett gravitationsfält).

Den tröga massan definieras av Newtons andra lag:

F = m1 a

Den tunga massan definieras av Newtons gravitationslag:

F = Gm2 M/r2

där gravitationskonstanten G är

G = 6.673 10-11 m3s-2kg-1

Vi får accelerationen i ett tyngdkraftsfält (attraherande massan M och avståndet från massan r) genom att sätta kraften i tröghetslagen lika med gravitationskraften. Vi får

a = F/m1 = (Gm2 M/r2)/m1

Om den tröga massan är lika med den tunga massan kan massorna elimineras och vi får

a = GM/r2

Accelerationen och därmed fallhastigheten är alltså oberoende av massan på den fallande kroppen!

Tyngdaccelerationen g vid jordytan blir

g = GM/R2 = 6.673*10-11*5.9736*1024/(6.371*106)2 = 9.82 m/s2

där M är massan och R radien.

Hur kan vi förstå det faktum att den tröga massan och den tunga massan är lika? Jo, det är en naturlig följd av Einsteins allmänna relativitetsteori. Denna säger att gravitation är ekvivalent ("samma som") acceleration. Låt oss anta att vi har ett gravitationsfält som ger tyngdaccelerationen g (nedåt). Detta är då ekvivalent med en hiss som har accelerationen g (uppåt). Vad händer om du släpper en lätt kula och en tung kula mitt i hissen? I förhållande till hissen (som ju accelererar uppåt) rör sig kulorna hela tiden med samma hastighet (eftersom de inte påverkas av någon kraft). Eftersom vi sade att acceleration och tyngdkraft är ekvivalenta, faller alltså alla kroppar med samma hastighet oberoende av deras massa. Man kan säga att Einstein i förhållande till Newton bytte ut en ekvivalensprincip (trög massa=tung massa) mot en annan (tyngdkraft=acceleration).

Från Newtons gravitationslag kan man härleda att planeter och satelliter rör sig i elliptiska banor, se Elliptic_orbit . I själva verket kan man härleda alla tre Keplers lagar, se fråga 12644 .

Se vidare Mass och länk 1.
/Peter E

Se även fråga 12833 och fråga 13930

Nyckelord: Newtons gravitationslag [12]; massa, trög/tung [4]; tyngdaccelerationen [16];

1 http://einstein.stanford.edu/STEP/

*

Ljud-Ljus-Vågor [13218]

Fråga:
Hej. Finns det någon bra demonstration/försök för en gymnasieklass att visa att ljushastigheten är konstant, oberoende av ljuskällans rörelse.
/Külvert J

Svar:
Külvert! Det är en ganska besvärlig fråga eftersom ljushastigheten i vakuum c är fundamentalt involverad i relativitetsteorin. Det korta svaret är nej det finns inget enkelt försök. Eftersom problemet är så grundläggande kan det vara på sin plats att diskutera det lite.

Att c är konstant oberoende av källans eller mottagarens rörelse är ett postulat (fundamentalt antagande) i Einsteins speciella relativitetsteori. Einstein grundade antagandet på resultatet av ett experiment där Michelson-Morley försökte påvisa etern. Etern vad det medium man trodde transporterade elektromagnetisk strålning. Resultatet av M-M försök var att de inte såg någon skillnad på olika riktningar trots att jorden rör sig i sin bana med en hastighet av 30 km/s. M-M tolkade resultatet så att jordens rörelse genom etern "tryckte ihop" jorden, så att det inte blev någon skillnad. Se vidare Michelson-Morley Experiment och Flash-animering på Michelson-Morley Experiment (Flash) . Observera att den senare visar vad som skulle hända om etern fanns.

Einstein omtolkade resultatet av M-Ms experiment så att han i stället antog att c är konstant oberoende av källans eller mottagarens rörelse. Från detta antagande härledde han en teori - den speciella relativitetsteorin - som i nästan 100 år visat sig stämma exakt med alla mätningar. Ljushastighetens konstans kan alltså anses väl etablerad.

Så etablerad att den internationella unionen för mått och vikt 1983 antog en ny definition av längdenheten meter, enligt vilken en meter är den sträcka som ljuset tillryggalägger i tomrum på 1/299 792 458 sekund. Därmed är ljushastigheten i vakuum definierad till exakt 299 792 458 m/s. Se vidare Is The Speed of Light Constant? , Speed_of_light och The Speed of Light .

M-Ms experiment med interferometrar i två vinkelräta riktningar visar (om man tolkar det så) att ljushastigheten är oberoende av mottagarens rörelse. Om man noggrannt observerar hur dubbelstjärnor rör sig i sina banor kan man även visa att ljushastigheten är oberoende av källans rörelse.

Så som du ser var det ingen lätt fråga du ställde . Tack Elisabeth och P-O för synpunkter !

Fotnot:

Experiment att bestämma ljushastigheten som är lätt att förstå men kräver lite utrustning finns i länk 1 nedan.
/Peter E

Se även fråga 3545 och fråga 12753

Nyckelord: elektromagnetisk strålning [21]; ljushastigheten [24]; relativitetsteorin, speciella [45]; #ljus [63];

1 http://www.csc.kth.se/~tomaso/ufu/ljusexperiment.html

*

Universum-Solen-Planeterna [14080]

Fråga:
Hej! Är det sant att om man kollar ut på Universum så ser det likadant ut i vilket riktning man än kollar? I så fall, hur kan man då upptäcka nya galaxer? Jag läste nyligen att ny galax har upptäckts. Den heter "Sombrero galaxy" och har cirka 2,400 stjärnor med hälften så liten massa som vår sol. P.S Isaac Newton hade skrivit många olika teorier om gravitationen men har han skrivit om hur den fungerar? Var kan man hitta informationen i såfall?
/Kir E, Tibble, Täby Centrum

Svar:
Ja, i medeltal ser universum likadant ut i alla riktningar. Det hindrar inte att det finns olika galaxer i olika riktningar.

2400 stjärnor skulle man knappast kalla en galax. Den enda Sombrero-galaxen jag känner till är M104, se bilden nedan (från länk 1). M104 är en jättegalax med c:a 2000 klotformiga stjärnhopar som innehåller hundratusentals stjärnor vardera.

Newtons graviationslag beskriver hur materia attraherar annan materia. Einstein beskrev i sin allmänna relativitetsteori gravitationen på ett annat sätt än Newton, men slutresultatet är nästan detsamma för de två beskrivningarna, se fråga 12834 .



/Peter E

Se även fråga 12834

Nyckelord: galax [28];

1 http://www.spitzer.caltech.edu/images/1419-ssc2005-11a-Spitzer-Spies-Spectacular-Sombrero

*

Kraft-Rörelse [14118]

Fråga:
Har svarandet upphört - eller frågandet? Brukar kolla in den senaste veckans svar och det tycks inte frågats/svarats på länge!!

Här en fråga: Ett tänt stearinljus placeras på ett cykelhjul som får rotera horisontellt. (Ljuslågan skyddas för drag.) Lågan ställer sig då in mot centrum för rörelsen. Förklaringen sägs vara att gasernas/lågans densitet är mindre än omgivningens. Betyder det att en låga på motsvarande sätt roterad i ett lufttomt rum inte skulle ställa sig inåt utan utåt?(Man kan ju ha ett arrangemang där syre tillförs under rotarionen.) Hur förklaras resultatet med hjälp av krafter och deras resultanter?
/Thomas Å, Märstagymnasiet, Märsta

Svar:
Thomas! Nej, varken frågandet eller svarandet har upphört! Vi som svarar tyckte emellertid vi var förtjänta av lite semester . Sedan är det också mycket att göra i början på terminen.

Vad gäller din fråga: Du har ett gravitationsfält riktat nedåt (gravitationen definierar ju normalt upp och ner) och en acceleration riktad in mot centrum av cykelhjulet. Gravitation och acceleration är till synes mycket olika begrepp, men i Einsteins allmänna relativitetsteori förenas de: instängd i ett litet utrymme (t.ex. en hiss) kan man inte avgöra om kraften fötterna påverkar golvet med orsakas av en massa eller av att hissen accelererar uppåt.

Detta kan man använda för att lösa ditt problem: se accelerationen mot centrum som en gravitationskraft riktad utåt. Sedan kan du sätta samman dessa två komposanter (en nedåt och en utåt) till en resultant utåt/nedåt.

I luft kommer lågan då att riktas uppåt/inåt eftersom lågans densitet är mindre än den omgivande luftens. I vakuum är lågans densitet större än omgivningens, så lågan bör riktas utåt/nedåt.
/Peter E

Nyckelord: relativitetsteorin, allmänna [33];

*

Kraft-Rörelse [14202]

Fråga:
Tja, om jag har förstått det rätt, är inte gravitationen en kraft utan det är dess krökning av rumtiden som gör att förmål krtesar kring massiva föremål. Det är bara därför att rumtiden är krökt som vi fattar det som att de ändrar riktning om de kommer i närheten av ett massivt föremål medans de uppfattar det som de åker rakt. Om detta stämmer varför vänder då en boll om man kastar upp den i luften? Det krävs ju en kraft för att byta till motsatt riktning...
/Johan M, St.Krullerskolan, Genéve

Svar:
Du får gärna betrakta gravitationen som en kraft, det är inte förbjudet! I fysiken använder man ofta modeller och analogier för att beskriva komplexa teorier. Einstein (egentligen först Minkowski) använde sig av en fyrdimensionell rum/tid i sin speciella relativitetsteori. Han kunde sedan mycket elegant beskriva gravitationen i den allmänna teorin som en krökning hos denna fyrdimensionella rum/tid. Jag tror inte det går att göra sig en bra och rättvisande bild av hur detta ser ut, man får vara nöjd med att den matematiska apparaten fungerar och återger observerade resultat.
/Peter E

Se även fråga 8419

*

Kraft-Rörelse [14685]

Fråga:
Vad finns det för bevis för relativitetsteorin?
/Veckans fråga

Ursprunglig fråga:
När kunde man eller kan man fortfarande inte fastställa relativitets teorin? Vilka olika upptäckter och bevis var man tvungna att få fram?
/Frida S, Kullagymnasiet, Höganäs

Svar:
Frida! Som fysikaliskt teori måste relativitetsteorin anses mycket etablerad. Detta gäller både relativitetsteorin, speciella och relativitetsteorin, allmänna . Se Special_theory_of_relativity#Consequences_derived_from_the_Lorentz_transformation och General_theory_of_relativity#Consequences_of_Einstein's_theory för några experimentella resultat som stöder den speciella och den allmänna relativitetsteorin.

Det universiella navigeringssystemet GPS (se Global_Positioning_System och nedanstående figur från Wikimedia Commons) med 24 satelliter i bana runt jorden på en höjd av 20000 km skulle helt enkelt inte fungera om man inte tog hänsyn till relativitetsteorierna.

Dels orsakar banrörelsen att den mycket exakta klockan i en satellit saktar sig 7 mikrosekunder per dygn pga den speciella relativitetsteorin. Eftersom satelliten befinner sig i ett svagare gravitationsfält går klockan 45 mikrosekunder per dygn snabbare. Nettokorrektionen 45-7=38 mikrosekunder per dygn appliceras genom att man justerar klockan att gå lite långsammare innan satelliten skickas upp. Man synkroniserar även alla klockorna med hjälp av klockor på marken. Se vidare länk 1.

Se vidare Special_relativity och General_relativity .


Tillägg 5/4/2011:

Uppskattning av effekterna

Konstanter:
Ljushastigheten: c = 3.00*108 m/s
Gravitationskonstanten: G = 6.675*10-11m3/(kg.s2)
Jordens massa: M = 5.974*1024 kg
Jordens radie: R = 6.37*106 m
GPS-satelliternas avstånd från jordens centrum (20000 km över jordytan): RGPS = 26.37*106 m

Speciell relativitet

Vi räknar från jordens centrum eftersom satelliterna går ganska högt och jordens rotationshastighet är liten (mindre än 500 m/s) jämfört med satelliternas hastighet.

Satellitens hastighet v ges av

mv2/RGPS = mMG/(RGPS)2

vilket blir

v = sqrt(MG/RGPS) = sqrt(5.97*1024*6.67*10-11/(26.367*106) = 3886 m/s

Klockan påverkas med g-faktorn (Special_relativity#Time_dilation_and_length_contraction )

g = sqrt(1-(v/c)2) = sqrt(1-(3886/300000000)^2) = 0.9999999999161055

Den relativa korrektionen blir

1 - g = 8.389*10-11

och korrektionen på ett dygn blir

8.389*10-11*60*60*24 = 7.25*10-6 s

eller c:a 7 mikrosekunder.

Allmän relativitet

Denna korrektion har att göra med att man måste bevara totala energin även i ett gravitationsfält, se fråga 16989 , stycket gravitationell rödförskjutning. För att bevara energiprincipen måste tiden i ett starkt gravitationsfält gå långsammare än i ett svagare. Tiden går alltså till synes snabbare i GPS satelliterna än på jordytan. Den relativa korrektionen ges av potentialskillnaden dividerat med viloenergin mc2 (m är satellitens massa som kommer att försvinna i slututtrycket).

Om gravitationspotentialen är U(r) så gäller genom integration av gravitationskraften

U(r) = -mMG/r

Ändringen i potentiell energi om vi går från jordytan till satellitbanan blir

DU = -mMG/RGPS -(-mMG/R) = -mMG(1/RGPS -1/R)

Med insatta värden blir

DU = m*4.714 107

Om vi dividerar detta med viloenergin mc2 får vi den relativa korrektionen till

5.237 10-10

På ett dygn blir korrektionen

5.237 10-10*24*60*60 s = 45.2 mikrosekunder

vilket stämmer bra med värdet ovan.



/Peter E

Nyckelord: relativitetsteorin, allmänna [33]; GPS [3]; relativitetsteorin, speciella [45];

1 http://www.astronomy.ohio-state.edu/~pogge/Ast162/Unit5/gps.html
2 http://metaresearch.org/cosmology/gps-relativity.asp

*

Kraft-Rörelse [14912]

Fråga:
När man räknar med relativitetsteorin och längd- och tidsdilation, hur vet man då att objektet som accelereras, av längddilationen förblir opåverkat i vinkelrät riktning mot färdriktningen?

Och hur vet man ens att objektet som accelereras för observatören krymper i längriktningen och att det inte ökar i längdriktningen för resenären istället?

Teoretiskt sätt liksom, Einstein kom väl på det här långt innan några praktiska experiment var gjorda?
/Kristofer K, Platengymnasiet, Motala

Svar:
Alla observationer tyder på att relativitetsteorin är korrekt. Det är emellertid svårt (omöjligt) att observera längdkontraktionen (se fråga 12892) direkt. Ett 3-dimesionellt objekt kommer t.ex. att roteras så att man kan "se om hörnet". Se länk 1 för en icke helt lättförståelig animering.
/Peter E

Se även fråga 12892

Nyckelord: relativitetsteorin, speciella [45];

1 http://faraday.physics.utoronto.ca/PVB/Harrison/SpecRel/Flash/ContractInvisible.html

*

Blandat [16048]

Fråga:
Grundläggande fysikaliska begrepp och definitioner
/Peter E

Svar:
Här samlas definitioner av några grundläggande fysikaliska begrepp


densitet (Density , densitet )

Densitet för ett ämne är massa per volymenhet (m/V)


massa (Mass , massa )

Massa är en fysikalisk storhet som anger ett objekts materieinnehåll. Massa är den egenskap hos materien (se materia ) som påverkas av gravitationskraften (tung massa) och som motsätter sig rörelseändringar (trög massa). Enligt Einsteins allmänna relativitetsteori är den tunga massan identisk med den tröga (ekvivalensprincipen). Enligt en annan ekvivalensprincip, i den speciella relativitetsteorin, är massan en form av energi: E=mc2 (E är energi, m massa, c ljushastigheten).

Jmfr tyngd och vikt. Se vidare massa, trög/tung .


statisk elektricitet (Static_electricity , statisk-elektricitet )

Fenomenet statisk elektricitet uppkommer efter att två föremåls ytor åtskiljs, efter att ha varit i kontakt med varandra. Anledningen är att elektronerna dras med olika kraft till olika material. Uppladdningen blir större när två föremål gnids mot varandra, därför att kontaktytan då ökar.

Ett klassiskt exempel på statisk elektricitet är den som bildas vid kamning av torrt, nytvättat hår. Då fastnar det negativt laddade elektroner på kammen, vilket ger upphov till en negativ laddnin laddning hos kammen och en positiv laddning hos håret.

Statisk elektricitet är farlig för många elektronikkomponenter, såsom minnen och IC-kretsar. I vissa branscher där man till exempel hanterar drivmedel och sprängämnen är statisk elektricitet ett kritiskt säkerhetsproblem.

Se vidare statisk elektricitet .


tryck (pressure , tryck )

Tryck är inom fysiken det samma som kraft per ytenhet. Det används normalt för gaser och vätskor. Tryckenheten i SI-systemet är 1 pascal (1 Pa = 1 newton per kvadratmeter, N/m2). Då pascal är en liten enhet, brukar ofta andra tryckenheter användas, till exempel: bar, mm Hg, atmosfär.


tyngd (Weight , tyngd )

Tyngd är den kraft som verkar på kroppar i ett gravitationsfält. Tyngden är proportionell mot föremålets massa med en proportionalitetskonstant som är tyngdaccelerationens lokala värde: tyngd = F = m*g.

Jmfr massa och vikt.


vikt (Weight , vikt )

Officiellt är ordet vikt synonymt med massa. Eftersom det dagliga språkbruket inte tydligt skiljer mellan vikt och tyngd och eftersom etymologiskt besläktade ord i andra språk som engelska weight och tyska Gewicht används för tyngd, rekommenderar SIS Standardiseringsgrupp att undvika ordet vikt (svenska Wikipedia).

Jmfr tyngd och massa.



/Peter E

Nyckelord: *fysikaliska definitioner [1];

*

Kraft-Rörelse [16263]

Fråga:
Hej hej, jag har en fråga om relativitetsteorin som jag för närvarande skriver ett arbete om i Fysiken.

Jag tror att jag har hittat en paradox när jag började räkna lite grann på tidsdilatationen. För det första;

Om man skulle befinna sig på ett tåg som rörde sig med en hastighet av 0.9999c under fem minuter, så skulle det för en utomstående observatör ha gått ca fem timmar, enligt t0=t/(sqrt(1-(v^2/c^2)) där t0 är observatörens tid, och t är tiden för personen på tåget.

t0=t/(sqrt(1-(v^2/c^2)) för t = 300s => 300/(sqrt(1-.9999^2) = 21 213.7338s ~ 5 timmar

om vi sedan släpper iväg ett tåg med halva hastigheten på samma sträcka, så skulle det ta 600s att färdas samma sträcka. Det leder till en tidsdilatation på

600 / sqrt(1 - (.49995^2)) = 692.797231 Alltså inte mer än 1.5 minuter!

Det betyder att för en utomstående observatör så skulle det långsammare tåget komma fram flera timmar före det snabbare!

Hur kan man förklara detta?

----

När jag tänkte lite på detta kom jag fram till ytterliggare en fråga, nämligen att ett ljusår (eller ljusminut, eller vilken annan avståndsbenämning som helst som bygger på ljushastigheten) ju är ett begrepp som är äldre än relativitetsprincipen. Betyder det att tidsdilatation inte tagits med i beräkningen av sträckan? För om så är fallet leder samma ekvation som tidigare till att en sträcka på 8 ljusår (jorden / sirius) skulle vara byggt på en tidsdilationerad bild av ljuset. Om man med samma tåg som tidigare åkte med en hastighet av 0.9999c så skulle man behöva färdas i endast en vecka för att tiden skall dilatationeras åtta år.

Hur går det ihop?

---

Frågan framför allt, vad har jag missat; jag tror knappast att jag hittat ett otäppbart hål i grunden för den moderna fysiken.

Tack.
/Johan S, Bergska Skolan, Finspång

Svar:
Hej Johan! Roligt att du funderar! Bra också att du inser att det krävs mycket för att kullkasta Einsteins speciella relativitetsteori. Eftersom ingen hittills lyckats (men många har försökt!) sedan 1905, så är det uppenbart att teorin vilar på ganska säker grund!

Låt oss först reda ut var ditt resonemang går fel och sedan lite om hur man skall uppfatta en fysikalisk teori och då speciellt relativitetsteorin.

I ditt första resonemang använder du dig av begreppet samtidighet, se fråga 3061 nedan och Relativity_of_simultaneity . Eftersom tidsdifferenser beror av hastigheten v och läget x (Lorentz-transformationen) så kan man inte utan vidare använda begrepp som "kommer först", "kommer efter", etc.

Din andra fråga har egentligen inget med relativitetsteorin att göra. Ett ljusår är en sträcka som ljuset tillryggalägger på ett år sett utifrån slutpunktens referensram. Ljushastigheten har varit känd med tillräcklig precision ganska länge, så det är inget problem att förvandla km till ljusår. En observatör som rör sig i förhållande till denna referensram kommer visserligen att ha en avvikande uppfattning om avståndet (längdkontraktion), men det förändrar inte det "verkliga" avståndet.

Den speciella relativitetsteorin utgår från två antaganden:

1 Naturlagarna är oberoende av rörelse med konstant hastighet

2 Ljushastigheten i vakuum c är densamma oberoende av observatörens rörelse

Dessa antaganden är rimliga med hänsyn till observationer, men de går inte att bevisa. Detta är en metod man ofta använder i vetenskapen: gör ett antagande och visa vad antagandet innebär vad gäller fenomen man kan observera. Om observationen skiljer sig från vad man väntat har man falsifierat teorin, och man får göra nya antaganden. Man har inte falsifierat relativitetsteorin, utan alla observationer stämmer med vad man väntar med utgångspunk från de två antagandena.

Detta är typiskt för naturvetenskapliga teorier: det går aldrig att bevisa att de är korrekta - endast att de är inkorrekta. De inkorrekta sorteras bort och lagras i vetenskapshistoriens skräpkammare. De som överlever blir, allteftersom nya typer av observationer visar sig stämma, mer och mer etablerade. Einsteins speciella relativitetsteori tillhör de mest etablerade fysikaliska teorierna.

Ibland kan steget mellan att formulera antagandena för en teori och att visa på vad teorin förutsäger vad gäller observationer vara svårgenomträngligt för en experimentalfysiker. Då får vi helt enkelt lita på att teoretikerna kan sin sak och att de formler de får fram är korrekta.

Den speciella relativitetsteorin ställer rimliga krav på matematikkunskaper - relativt enkel algebra räcker. Svårigheten är att vissa av resultaten av teorin står i strid med intuitionen.

Den allmänna relativitetsteorin (se General_relativity ) kräver däremot mycket avancerad matematik som få behärskar. Här får man helt enkelt nöja sig med antagandet (ekvivalens mellan acceleration och gravitation) och att resultatet av alla observationer till fullo stöder teorin.

Är då relativitetsteorierna de slutgiltiga teorierna? Nej, det är de inte, vi saknar bland annat en förening med kvantmekaniken. En ny teori kommer att omfatta relativitetsteorierna men utökas till att även ta hänsyn till kvantmekaniska fenomen.

Det finns en mycket omfattande och bra artikel i engelska Wikipedia: Special_theory_of_relativity . Se även Speciella_relativitetsteorin och fråga 16270 nedan.
/Peter E

Se även fråga 3061 och fråga 16270

Nyckelord: relativitetsteorin, speciella [45]; fysik, förståelse av [17]; relativitetsteorin, allmänna [33];

*

Blandat [16415]

Fråga:
Har ni några konkreta tips som skulle kunna hjälpa mig i kampen om att bli bra på Fysik?
/Veckans fråga

Ursprunglig fråga:
Hej!

Jag älskar kemi, matte och fysik!

Matte och Kemi klarar jag mig bra i (MVG/VG i).

Men Fysik är mycket svårt och jag vill verkligen få ett bra betyg i Fysik A som jag läser nu.

Har ni några konkreta tips som skulle kunna hjälpa mig i kampen om att bli bra på Fysik och att få MVG/VG?
/Pelle S, S:t petri, Malmö

Svar:
Hej Pelle!

Trevligt att du tycker om fysik! Bara att du har en positiv attityd till fysikämnet är en god början! Fysik är egentligen inte svårt - problemet är nog att det omfattar så mycket. Det är många begrepp, definitioner och lagar man måste ta till sig, men man skall inte överdriva nyttan av att kunna en massa formler utantill - det är bättre att göra fomelsamlingen till sin vän.

Fysiken blir emellertid lättare med tiden - man börjar så småningom se att det finns ett mycket begränsat antal grundläggande lagar. Det är emellertid inte så lätt att tillämpa dessa eftersom fysikaliska system ofta är mycket komplexa. Man får då arbeta med förenklade fysikaliska modeller, se fysikalisk modell . Ofta kan man inte få fram en analytisk lösning (en enkel formel), utan man måste arbeta med parametriseringar och datorsimuleringar.

Ett problem som jag stöter på ofta är att man säger att fysik är svårt för att man inte kan "förklara" naturlagarna. Varför attraherar t.ex. två massor varandra. Vi kan beskriva fenomenet utmärkt med Newtons gravitationslag eller om vi så vill med Einsteins allmänna relativitetsteori, men i djupare mening begriper vi ingenting , se fysik, förståelse av .

När man kommit förbi gymnasiefysiken (som måste försöka täcka det mesta av det viktigaste av den klassiska fysiken) är fysik mycket mer intressant. Dels har man olika inriktningar (t.ex. elementarpartikelfysik, atomfysik, kärnfysik, fasta tillståndets fysik) och dels kan man alltefter talang välja teoretisk inriktning, experimentell inriktning eller tillämpningar. De senare kan antingen vara andra naturvetenskaper (biologi, geologi etc) eller rent tekniska tillämpningar t.ex. att framställa miljövänliga lampor.

Det är ett problem med den traditionella fysikundervisningen att den oftast följer den historiska utvecklingen. Det betyder att i början får man bara studera klassisk fysik, t.ex. mekanik och ellära. Detta upplevs av många som tråkigt, så fysiken får ett oförtjänt dåligt rykte. Det vore bättre om man på ett tidigt stadium kunde blanda in nutida fysik (se fråga 14232 för definitioner av begreppen).

Man måste vänja sig vid att fysik är något annat än matematik. I matematik utgår man från ett antal axiom och bygger en struktur från dessa. I fysik utgår man från resultatet av experiment och observationer och försöker se samband, ofta med användning av matematik (delar av matematiken har faktiskt utvecklats för att lösa fysikaliska problem) och förenklade modeller. För mig är just förmågan att modellera komplicerade system en typisk egenskap en bra fysiker skall ha.

Här finns några frågor som är relevanta: fysik, nytta med , forskningskarriär , vetenskaplig metod och fysik . Wikipedia-artiklarna Physics och Fysik är mycket bra.

Det är svårt att ge konkreta råd vad du skall göra för att få bra betyg, men om du klarar matematik och kemi så borde det inte vara något problem. I övrigt tycker jag du skall välja en del av fysiken för specialstudier, men samtidigt läsa populärvetenskapliga böcker och artiklar om all naturvetenskap. Forskning i dag är ofta gränsöverskridande, alltså forskning som involverar mer än ett klassiskt ämne. Lycka till i dina studier!
/Peter E

Nyckelord: fysik [10]; matematik i fysik [6];

*

Blandat [16588]

Fråga:
Hej jag skriver just nu ett arbete om fysikens historia och fysikens innebörd för samhället idag. Pga detta så undrar jag lite om vad som har hänt med fysiken under 1900-talet samt vilka betydelser det har för oss idag. (Det räcker med att ni tar upp 4 exempel på teorier eller "fysikuppfinningar" som uppkom under 1900-talet)

Bästa hälsningar Fysikälskaren Ellen :)
/ellen s, erik rydberg, stockholm

Svar:
Bra försök Ellen, men eftersom du älskar fysik så vill du väl skriva arbetet själv ! Jag kan bara ge dig några tips.

Wikipedia-artikeln History_of_physics är bra och har många länkar. Artikeln fysik i Nationlencyklopedin är också bra. Man får även anta att de upptäckter som betytt mest för samhället belönats med nobelpris i fysik. Så ett tips är att botanisera bland fysikprisen, se The Nobel Prize in Physics - Laureates . Speciellt för de senare priserna finns det mycket bra artiklar.

Fysiken som utvecklades i början av 1900-talet är grunden för all nutidsfysik. Det var Einsteins relativitetsteorier och kvantmekaniken. De upptäckter som påverkat samhället mest är nog fission (Nuclear_fission ), transistorn (Transistor , nobelpris 1956) och den integrerade kretsen (Integrated_circuit , nobelpris 2000). Upptäckterna bakom fysikprisen 2007 och 2009 har också betytt mycket för utvecklingen av datorer och kommunikation.
/Peter E

Se även fråga 8405

Nyckelord: fysik [10];

1 http://fragelada.fysik.org/links/search.asp?keyword=vetenskapshistoria

*

Kraft-Rörelse [16839]

Fråga:
Hej! Jag undervisar i fysik och undrar hur man någorlunda enkelt kan förklara att ljushastigheten är den största hastigheten som existerar.
/Lennart N, Vimarskolan, Vimmerby

Svar:
Lennart! Som mycket i naturvetenskap kan man inte förklara det. Det är helt enkelt ett experimentellt faktum att inget kan färdas snabbare än ljushastigheten i vakuum. Einstein antog att ljushastigheten var oberoende av källans och detektorns rörelse, och utvecklade från detta den speciella relativitetsteorin. Denna teori har stöd i ett stort antal observationer.

Det har framförts teorier att man kan överskrida ljushastigheten i vakuum, se fråga 15804 nedan, men det finns inget allmänt accepterat experimentellt stöd för detta.
/Peter E

Se även fråga 15804

*

Universum-Solen-Planeterna [16874]

Fråga:
Hej! Jag har en fråga angående Big Bang och Einsteins relativitetsteori. Einstein trodde att universum var statiskt. Enligt hans lag om gtravitationen vill all materia dras till varandra. Han införde den kosmologiska konstanten som en "antigravitationskraft". Detta för att inte universums materia skulle dras samman och krocka. När Hubble upptäckte att galaxer rör sig från varandra och bevisade att universum expanderar gav Einstein upp sina teorier om ett statiskt universum. Han strök den kosmologiska kanstanten och kallade den sitt livs största misstag.

Min fråga är: Varför strök Einstein den kosmologiska konstanten ur sina beräkningar? Om Galaxerna rör sig från varandra och dessutom fortare ju längre från varandra de kommer måste något annat än bara gravitationen påverka materian.
/Ellinor C, Tranängsskolan, Limmared

Svar:
Ellinor! Allt du säger är korrekt. Det enda är att Einstein när han gav upp den kosmologiska konstanten bara visste - från Hubbles rödförskjutning - att universum expanderade. Han behövde alltså inte någon repulsion som gav ett statiskt universum. Om väl expansionen satts igång (vid big bang) så fortsätter den i stort sett obehindrat om bara universums densitet är tillräckligt låg.

I dag har vi data som antyder att expansionen accelererar, se figuren i fråga 7258 nedan. Effekten syns bara på mycket stora avstånd - mycket större än de som Hubbles data omfattar. Denna acceleration kräver en kraft, och det är för att förklara denna repulsiva kraft som den kosmologiska konstanten har återinförts.
/Peter E

Se även fråga 7258

Nyckelord: universums expansion [16];

*

Ljud-Ljus-Vågor [16989]

Fråga:
Har fotonen massa?
/Veckans fråga

Ursprunglig fråga:
Enligt relativitetsteorin har fotonen ingen massa, pga. att den rör sig med ljushastigheten. Samtidigt påverkas fotoner av gravitationen (avböjning av ljus som passerar nära en massa, gravitationell rödförskjutning).

Jag har läst att man har visat experimentelt att ljuset ändrar sin frekvens(minsking av energi) när den färdas från jorden. Tycker att det låter konstigt!

Min lärare sa att "Fotonen har massan noll och det är därför den rör sig med ljusfarten. Alla objekt som har massa rör sig med farter som är mindre än ljusfarten." Hur kan man då förklara det med experimentet där fotonen kan påverkas av gravitationsfält??

Det är helt förvirrat för mig då jag läser att fotoner inte har någon massa men den kan ändå ha partikelegenskaper. Hur kan det komma sig? Och vad betyder egentligen dessa experiment som har vissat att fotoner påverkas av gravitationskraften och att den har rörelsemängd (compton spridning).
/Ali Z, borgarskolan, malmö

Svar:
Ali! Det var många svåra frågor. Låt oss börja med partikel-egenskaper. Jag tycker inte man skall föreställa sig en foton varken som en partikel eller en våg. En foton är en foton som lånat egenskaper både från partiklar och vågor.

Fotonens massa: Fotonen har energin E=hv. Eftersom energi och massa är ekvivalenta (E=mc2), så har fotonen massa. Man kan emellertid inte tala om fotonens vilomassa eftersom begreppet en stillastående foton saknar mening.

Experimentella bevis för att fotonen saknar vilomassa kommer bland annat från det faktum att den elektrostatiska kraften (Coulombs lag, som ju förklaras genom ett utbyte av virtuella fotoner) varierar som 1/r2 En utbytespartikel med ändlig vilomassa hade givit ett annat avståndsberoende.

Se fråga 16939 för mer om historien bakom fotonbegreppet och länk 1 för en mer detaljerad framställning. Wikipedia-artikeln Photon är mycket bra, medan den svenska versionen är OK men inte särskilt omfattande: Foton .

Fotoner påverkas på två sätt av gravitationsfält:

1 Avböjning, t.ex. vid passage nära solen. Detta har behandlats ganska detaljerat i fråga 16021 .

2 Gravitationell rödförskjutning.

Till skillnad från avböjning så kräver faktiskt en tillfredsställande behandling av gravitationell rödförskjutning bara enkel klassisk fysik och speciella relativitetsteorins E=mc2. Börja med att studera den enkla animeringen under länk 2!

Den potentiella energin hos elektronerna i övre läget är (mgh)

2me*g*H

Om vi förlänger med c2 får vi

2mec2*(g*H/c2) = Efoton*(g*H/c2)

Vi får det relativa skiftet för höjdskillnaden 22.5 m (ref. 1):

DEfoton/Efoton = gH/c2 = 9.803*22.5/(2.998*108)2 = 2.454 10-15

Det relativa skiftet har uppmätts (ref. 1) med hjälp av mössbauerspektroskopi (se nedan) för 14.4 keV fotoner från 57Co-sönderfall. Det uppmätta resultatet 2.451 10-15 (med c:a 1% osäkerhet) är i bra överensstämmelse med detta.

Observera att det enda antagande vi gör är energins bevarande, vilket är en av fysikens grundläggande och mest etablerade lagar.

Observera även att man kan se skiftet till större våglängd (och därmed lägre frekvens) när fotonerna går uppåt är ekvivalent med att klockan går långsammare i ett starkare gravitationsfält.

Liknande experiment med hjälp av satelliter som sänder ut en mycket välbestämd frekvens har bekräftat Einsteins teori med en noggranhet bättre än 1 del på 104.

Se vidare General_relativity , Gravitational_redshift och Pound-Rebka_experiment .

Se även http://fy.chalmers.se/~f1xjk/FysikaliskaPrinciper/FOREL.lp2/F16/F16.html

Mössbauerspektroskopi

Mössbauer-effekten är rekylfri emission och absorption av gammastrålning från atomkärnor. När en atomkärna utsänder ett gammakvantum förloras i en del av energin till kärnans rekyl liksom vid absorption i en absorberande kärna. Detta eftersom både energi och rörelsemängd måste bevaras i processen.

I mössbauereffekten elimineras förlusterna dels genom att de radioaktiva kärnorna sitter i en kristall som tar upp rekylen, så att emission och absorption kan ske vid samma energi och dels genom att man kan kompensera energiförlusten genom att låta den utsändande kärnan röra sig.

Eftersom rekylen är mycket liten räcker det med en mycket måttlig hastighet på några mm/sekund, se nedanstående bild där det lilla diagrammet är en plot av ett mössbauerspektrum med hastighet på den horisontella axeln och observerad intensitet på den vertikala. Dippen i spektrum reflekterar det exciterade tillståndets vidd DE. Vidden är relaterad till tillståndets livslängd enligt Heisenbergs obestämdhetsrelation :

DDt = h / 4p ~ 10-34 Js

Se vidare Mössbauer_spectroscopy och mössbauer-effekten . Se även fråga 14685 . ____________________________________________________________
1 Pound and Snider, Physical Review Letters Vol 13, 18 (1964) 539



/Peter E

Nyckelord: relativitetsteorin, allmänna [33]; #ljus [63];

1 http://www.desy.de/user/projects/Physics/ParticleAndNuclear/photon_mass.html
2 http://fragelada.fysik.org/reltest1/frame.htm

*

Kraft-Rörelse [17264]

Fråga:
Hej! Jag har läst en artikel illustrerad vetenskap om tyngdkraft och där i fanns en formel:

F = (G*m_1*m_2)/r^2 + A/r^4

Jag vet att " F = (G*m_1*m_2)/r^2 " är Newtons formel om tyngdkraft. Jag läste att A är ett tal man kan få ut ifrån einsteins relativitetsteori.

Nu är då frågan: Vad står A för?

När jag kollade på nätet efter einsteins relativitets teori så hittade jag formeln:

R_uv - 1/2 Rg_uv = 8piGT_uv

Utläst: R nedsänkt i uv minus hälften av R gånger g nedsänkt i uv är lika med 8 pi gånger G gånger T nedsänkt i uv.

Det stod också: "den vänstra sidan beskriver universums krökning och den högra beskriver fördelningen av massa och energi i universum".

Till frågan: Har den sista formeln något att göra med Aet i den första formeln? Eller är det så enkelt att A ÄR den sista formeln??

Om (inte) det stämmer, kan ni inte förklara den andra formeln mer ingående i alla fall? :)
/Marcus B, Östlyckeskolan, Alingsås

Svar:
Marcus! Nej, A-termen i första formeln har inget med allmänna relativitetsteorin att göra. A-termen ger, eftersom den innehåller 1/r4, en avvikelse från den klassiska teorin för små avstånd. Om A är negativt skulle den kunna ge repulsion när r är tillräckligt litet. Det finns ingen experimentell bekräftelse på att A-termen behövs.

Einsteins gravitationsteori är matematiskt mycket komplicerad (det tog lång tid för Einstein att formulera den eftersom han inte från början inte behärskade matematiken) och den utgör en 7.5 hp kurs på avancerad universitetsnivå. Men man kan i varje fall konstatera att ekvationen är vacker!
/Peter E

*

Kraft-Rörelse [17325]

Fråga:
Vem kom på att två föremål oavsett vikten faller till marken samtidigt?
/Veckans fråga

Ursprunglig fråga:
vem som kom på att två föremål oavsett vikten faller på marken samtidigt?
/Frank M, Nacka, stockholm

Svar:
Frank! Det var Galileo Galilei som först utförde experiment och resonerade om detta, se Galileo_Galilei#Physics . Einstein använde ekvivalensen mellan acceleration och gravitation som ett grundantagande för sin allmänna relativitetsteori. Av detta följer direkt att alla kroppar faller lika snabbt i ett tyngdkraftfält. Man kan alltså byta ut ett tyngdkraftfält med en acceleration i motsatt riktning.

Låt oss säga du står i en hiss som befinner sig på marken (se nedanstående bild från länk 1). Du släpper ett föremål som faler nedåt. På grund av ovanstående ekvivalensprincip kan vi byta ut tyngdkraftfältet mot en lika stor acceleration uppåt. Det är uppenbart i detta fall (till höger i figuren) att fallhastigheten är helt oberoende av massan hos föremålet eftersom det ju står stilla (påverkas inte av någon kraft) medan hissen och du (som står på golvet) accelererar uppåt.

Se fråga 13663 för en kul demonstration.

Tillägg om ekvivalensprincipen

Engelska Wikipedia definierar ekvivalensprincipen som (Equivalence_principle ):

In the physics of general relativity, the equivalence principle is any of several related concepts dealing with the equivalence of gravitational and inertial mass, and to Albert Einstein's observation that the gravitational "force" as experienced locally while standing on a massive body (such as the Earth) is actually the same as the pseudo-force experienced by an observer in a non-inertial (accelerated) frame of reference.

Ekvivalensprincipen förekommer även i en svagare form i klassisk newtonsk fysik: ekvivalensen mellan tung och trög massa, dvs att massan m som förekommer i uttrycket

F = m a

är samma som förekommer i Newtons gravitationslag

F = G M m/r2

dvs accelerationen a ges av

a = G M/r2

oberoende av m.

Av detta följer att alla föremål faller med samma acceleration i ett gravitationsfält (bortsett från luftmotståndet).

Länk 2 innehåller Galileis teoretiska argumentation för ekvivalensprincipen. Se A Cultural History of Gravity and the Equivalence Principle för diskussion om tidigare funderingar (den Bysantinske filosofen Iohannes Philiponus).



/Peter E

Nyckelord: relativitetsteorin, allmänna [33];

1 http://www.astronomynotes.com/relativity/s3.htm
2 http://arxiv.org/abs/1208.5137

*

Kraft-Rörelse [17609]

Fråga:
Varför är ljusets hastighet i kvadrat i Einsteins formel? Eftersom inget kan gå fortare än ljusets hastighet i vakuum, hur kan det då gå i kvadrat?
/Lage S

Svar:
Det grundläggande svaret är att sådan är naturen. Einstein härledde sambandet E=mc2 från sin speciella relativitetsteori i en artikel från 1905, se fråga 12753 .

Viloenergin E0=m0c2 kan göras troligt från uttrycket för relativistisk massa, se fråga 14250 .

Om man antar att totala energin E beror av m och c kan man se genom en dimensionsanalys att E=mc2:

Energin E i vänsterledet har dimensionen J. Högerledet har dimensionen

kg*(m/s)2 = kg*m/s2*m = N*m = J

Exponenten för m måste alltså vara 1 och exponenten för c vara 2 för att dimensionerna skall stämma.

c2 har inget att göra med att något rör sig. c2 i detta sammanhanget är bara som vilken naturkonstant som helst, t.ex. Newtons gravitationskonstant G, se fråga 12834 .
/Peter E

Nyckelord: relativitetsteorin, speciella [45];

*

Kraft-Rörelse, Partiklar [17622]

Fråga:
Myonernas liv är ett bevis på att Einstein faktiskt hade rätt. Förklara hur och varför Myonernas liv är ett bevis för att Einsteins relativitetsteori är korrekt.

Sätt in svaret i ett generellt vetenskapstänkande. Alltså motivera ditt svar genom att sätta in det i en allmän naturvetenskaplig vetenskapssyn.

Hur ska jag på enklaste sätt förklara svaret för mina elever?
/alex a, katrinelund, göteborg

Svar:
Alex! Se fråga 2697 och 1074 .

Man kan aldrig bevisa att en teori är korrekt - man kan bara bevisa att den är i överensstämmelse med experiment eller inte. I det senare fallet får man förbättra teorin.

Vad gäller andra delen av din fråga så är den väl omfattande för frågelådan. Se fråga 13406 och länkar därifrån för mer om detta.
/Peter E

*

Partiklar [18067]

Fråga:
Man har på CERN mätt att neutriner rör sig men en hastighet överstigande ljushastigheten. Är det verkligen möjligt?
/Veckans fråga

Ursprunglig fråga:
Man har på CERN mätt att neutriner rör sig men en hastighet överstigande ljushastigheten. Är det verkligen möjligt?
/Sven D

Svar:
Du refererar till ämnet för pressreleasen under länk 1. Felet i den urspungliga mätningen är nu lokaliserat, se Hade Einstein fel? , så problemet med neutriner som rör sig snabbare än ljuset är löst.

En av forskningsgrupperna på CERN (OPERA-kollaborationen) har mätt löptiden hos neutriner från CERN till gruvan Gran Sasso 73 mil ner i Italien. Totalt har man detekterat c:a 16000 neutriner. Resultatet är att man får en hastighet som är lite överstigande ljushastigheten i vakuum c. Resultatet är signifikant med 6 standardavvikelser.

Löpsträckan på 73 mil (löptid 2.4 millisekunder med ljushastigheten) bestämdes med GPS med en precision på 20 cm. Tidssynkroniseringen gjordes även den med GPS och hade en precision av 1 ns. Totala osäkerheten i tidsdata uppskattades till 10 ns, och den uppmätta effekten var 60 ns. Denna diskrepans på 60 ns motsvarar t.ex. ett fel i avståndsmätningen på 60*10-9*3*108 = 18 m.

Vad gäller mätningen av löpsträckan var den största svårigheten att mäta sträckan inne i tunneln i gruvan Gran Sasso - GPSen fungerar dåligt under 1400 m berg! Man kan i mätningarna (som pågått sedan 2009) tydligt se effekter av kontinentaldriften och ett hopp i samband med en jordbävning i Italien 2009, se nedanstående figur.

Man gav ett webbseminarium från CERN fredagen den 23 aeptember 2011, länk 2. Där presenterade man resultatet och metoderna mycket bra och detaljerat.

Kommentarer:

* De flesta fysiker tror nog att mätningen är felaktig, men än så länge finns ingen bra förklaring. Troligaste orsaken är att sträckan är felmätt eller att tidssynkroniseringen CERN-Gran Sassio är felaktig.

* Einsteins speciella relativitetsteori förbjuder inte expicivt att partiklar rör sig med överljushastighet. Vad den förbjuder är att en partikel accelereras upp till ljushastigheten, det skulle kosta oändlig energi. Enda problemet är att partiklar som rör sig snabbare än ljuset (hypotetiska partiklar som kallas tachyoner, se Tachyon ) får imaginär massa. Se även fråga 15804 .

* OPERA-kollaborationen har helt följt god vetenskaplig procedur:
- Analysen har gjorts "blind" så att man inte skulle kunna påverkas av det väntade resultatet.
- Arbetet presenterades utförligt på ett öppet seminarium och i ett publicerat preprint.

* Det är inte första gången man försökt mäta neutriners hastighet, men man har inte fått fram definitiva värden. Den enda signifikanta observationen hittills är neutriner från supernovan SN 1987A, se fråga 125 och Supernova_1987A . Denna mätning visar att neutrinernas hastighet är mycket nära ljushastigheten. Att neutrinerna kom fram några timmar före ljuset kan förklaras att stjärnan är transparent för neutriner tidigare än den är transparent för ljus. Avståndet till SN 1987A är 168000 ljusår. Med den uppmätta tidsdifferensen skulle neutrinerna ha kommit fram

(60*10-9/2.4*10-3)*168000 = 4.2 år före ljuset.

Neutrinerna skulle alltså kommit fram flera år innan supernovan upptäcktes, inte några timmar. Tyvärr var neutrinodetektorerna inte igång då (Kamiokande II började ta data 1985, se (Kamioka_Observatory ).

_____________________________________________________________________

Fotnot: Nedanstående meddelande distribuerades i samband med pressreleasen. Det får anses helt unikt att en officiell deklaration från CERN går ut tillsammans med en pressrelease.

Dear Colleagues,

As usual, I am sending you this CERN press release before we issue it to the media. Unusually this time, however, I feel that it needs a few words of introduction. The OPERA collaboration has measured the time of flight of neutrinos sent from CERN to Gran Sasso, along with the distance they cover. These measurements appear to show that the neutrinos are travelling faster than light. When a collaboration makes a surprising observation such as this and is unable to account for it, the ethics of Science demand that the results be made available to a wider community, to seek scrutiny and to encourage independent experiments. That's why when the spokesperson of the OPERA collaboration asked me whether they could hold a seminar here, I said yes. Given the potential impact of such a measurement, I felt it important for CERN formally to make its position clear. That's the reason for the cautiously worded statement we're sending to the media today.

Best regards,

Rolf Heuer

Se även Hade Einstein fel? där det längst ner finns en uppdatering av statusen för experimentet.



/Peter E

Nyckelord: relativitetsteorin, speciella [45]; neutrino [19]; ljushastigheten [24]; nyheter [11]; SN 1987A [4];

1 http://press.web.cern.ch/press/PressReleases/Releases2011/PR19.11E.html
2 http://cdsweb.cern.ch/record/1384486

*

Blandat [18296]

Fråga:
Naturvetenskapen beskriver modeller av sådant vi aldrig sett. Hur kan man det?
/Veckans fråga

Ursprunglig fråga:
Naturvetenskapen beskriver modeller av sådant vi egentligen aldrig sett.

Hur kan man veta något om universums byggnad när man aldrig varit utanför vårt solsystem? Hur kan man veta något om jordens innandöme när det djupaste borrhålet i jordskorpan är 12 km? Hur kan man göra en modell av atomen när ingen har sett hur den ser ut?

Vad menas med att något har besvarats objektivt? Vad är motsatsen? Kan man någonson vara helt objektiv? Vad skulle kunna förhindra att något är helt objektivt?

Kan man vara naturvetare och samtidigt tro på gud?
/sofia k, skolstaden, olympia, Helsingborg

Svar:
Hej Sofia! Mycket bra men svåra frågor du ställer!

Naturvetenskap

I naturvetenskap har man för det första några grundregler som kallas vetenskaplig metod, se fråga 13406 . I fråga 14237 diskuteras god vetenskaplig metod och dess motsats, pseudovetenskap.

Vad gäller fysikaliska modeller så är de förenklade bilder av verkligheten som hjälper oss att förstå världen omkring oss. Modeller är inte detsamma som verkligheten, men en bra modell skall beskriva observationer så bra som möjligt.

Några exempel på fysikaliska modeller

Grunden för alla fysikaliska modeller är observationer och experiment. För kosmologi kan vi t.ex. observera den kosmiska bakgrundsstrålningen, mäta heliumhalten i stjärnor med spektroskopi, se att universum expanderar med dopplereffekten och mäta ljusstyrkan hos supernovor. Allt detta tillsammans skall passa in i en acceptabel modell för universums utveckling och struktur.

För jordens inre har man skaffat sig kunskap genom att studera sesmiska vågor vid t.ex. jordbävningar, se fråga 1052 . Dessutom kan man ju analysera material som kommer ut vid vulkanutbrott.

Bohrmodellen, fråga 13733 , beskriver atomen mycket förenklat. Den är en bra utgångspunkt men ger intrycket att elektroner rör sig i fixa banor.

Vätskedroppsmodellen är en enkel modell av atomkärnan som trots sin enkelhet förklarar förvånansvärt många egenskaper hos atomkärnor, se Vätskedroppsmodellen .

För att försöka förstå problemet med global uppvärmning använder man enkla och mycket sofistikerade klimatmodeller, se fråga 16846 .

För gravitation använder man fortfarande oftast Newtons beskrivning även om Einsteins allmänna relativitetsteori är mer grundläggande och mer korrekt i vissa fall. Det är inte ovanligt att man använder olika modeller för samma fenomen, aningen för att räkningarna blir enklare eller för att man bara är intresserad av en viss aspekt hos fenomenet.

Objektivitet

Objektivt är något som inte beror på observatören, och detta är något man strävar efter i naturvetenskap. Om du står på marken och släpper en boll, så kan nog alla vara överens om att bollen faller till marken. Om du däremot ställer frågan "vilken färg har bollen?", så kan du tänkas få olika svar eftersom hur man uppfattar färger är något subjektivt (motsatsen till objektivt).

Hundraprocentig objektivitet är mycket svårt att åstadkomma eftersom man redan när man bestämmer hur ett experiment eller en observation skall utföras, så har man infört ett mått av subjektivitet.

Religion

Ja, det finns många exempel på naturvetare som trott på Gud. Einstein är ett exempel.

Stephen Hawking är däremot ett exempel på en forskare som anser att det inte finns något behov av en gud, se fråga 17334 . Carl Sagan hade en ganska avvisande attityd till religion.

Många naturvetare föreställer sig inte en personlig gud utan något mer abstrakt, kanske t.o.m. att man uppfattar naturlagarna som ett gudomligt väsen.

Se även Vetenskapsteori , länk 1 och Fysik#Teori_och_experiment .
/Peter E

Nyckelord: vetenskaplig metod [18]; fysikalisk modell [12];

1 http://www.imit.kth.se/courses/2B1120/lecture/F1-modeller-energi.pdf

*

Ljud-Ljus-Vågor [18623]

Fråga:
Hej! Jag undrar om det är bevisat att ljusets FART inte påverkas av gravitation. Jag menar Einstein antog väl bara det, utan att egentligen veta.
/Jonas G, Nyfikenskolan, uppsala

Svar:
Ljushastigheten är alltid densamma. Detta är det grundläggande antagandet i relativitetsteorin, som ju fungerat ganska bra.
/Peter E

*

Kraft-Rörelse, Universum-Solen-Planeterna [18713]

Fråga:
Hej! Håller Einsteins relativitetsteori "i praktiken"? En tidningsartikel i SvD 26/5 meddelar att med ett nytt stort teleskop i Australien/Sydafrika hoppas man bevisa detta. Är inte detta redan gjort? (I samma artikel nämns att "man hoppas se vad som inträffade för nära en miljard år sedan, nära den tidpunkt då vårt universum föddes". Jag undrar!)
/Thomas Å, Knivsta

Svar:
Thomas! SvD artikeln (länk 1) var ganska vag och handlade mest om slagsmålet om placeringen av radioteleskopsystemet SKA (Square Kilometre Array).

Projektets webbsajt (länk 2) innehåller mer information. Ja, Einsteins allmänna relativitetsteori är ganska väl bevisad tidigare, se t.ex. fråga 473 . Man vill emellertid undersöka teorin för ännu högre gravitationsfält. Detta är dessutom bara ett av många planerade projekt.
/Peter E

1 http://www.svd.se/nyheter/utrikes/delat-teleskop-efter-seg-kamp_7231225.svd
2 http://www.skatelescope.org/

*

Kraft-Rörelse [18882]

Fråga:
Hej, jag går andra året på naturvetenskaps programmet. Jag har en fråga, finns det planer på att testa om Einsteins tvillingparadox stämmer. I teorin så skulle man kunna bygga en större accelerator som den som finns i Cern, en som skulle kunna skicka runt med hjälp av magnetism precis som i cern en "plåtburk" med en organism som har relativt låg livslängd. Sedan skulle man kunna accelerera så långsamt långsamt att g-krafterna inte på verkar organismen så mycket upp till näst intill ljusets hastighet. Hade organismen skulle man kunna jämföra om det faktiskt har skett en förändring i ålder mellan de två. Om organismen har färdats i ljusets hastighet tillräckligt länge. I teorin skulle man då bevisa om tvillingparadoxen stämmer eller ej. Skulle detta i teorin funka? Och finns det några planer på att i framtiden testa detta? tacksam för svar Ellen
/Ellen B, De la Gardiegymnasiet, Lidköping

Svar:
Hej Ellen! Att testa tvillingparadoxen med levande organismer ligger långt ifrån vad vi kan åstdkomma i dag - effekterna är alldeles för små för de hastigheter och avstånd vi kan åstadkomma. Vi får nöja oss med mycket exakta klockor, och lita på att tiden som en klocka mäter är samma som den tid får en organism att åldras, se Twin_paradox#The_equivalence_of_biological_aging_and_clock_time-keeping .

Man har utfört mycket noggranna mätningar med klockor och funnit att tidskillnaden är exakt den som den speciella relativitetsteorin förutsäger.

Se vidare nedanstående länk till andra svar om "tvillingparadoxen". Som synes är det ingen paradox även om vi kan tycka att resultatet är förvånande.

Se även Tvillingparadoxen , Hafele-Keating-experimentet , Twin_paradox (varning: mycket avancerad) och Hafele–Keating_experiment .
/Peter E

Nyckelord: tvillingparadoxen [5];

*

Kraft-Rörelse [18950]

Fråga:
Hej. Jag har läst på mycket om Einsteins relativitets teori plus den allmäna. Det är avancerade saker men en sak jag inte förstår är tvillingparadoxen, som i mitt fall krångar till hela relativitetsteorin. Tvillingparadoxen har alltid förklarats med att den ena personen accelererar från en viss punkt. Min fråga är:Vad händer om person A och B båda accelererar ifrån varandra från en punkt samtidigt? Och sedan bromsar in och accellererar tillbaka mot samma punkt igen? Då har båda personerna utsätts för samma gravitation/kraft. Tvillingparadoxen förklaras ju alltid med att den ENA personen accelererar och därmed töjs tiden ut för DEN personen. Men om BÅDA personerna utsätts för samma kraft/accellerlation så finns det ju ingen skillnad i deras referenssystem. Jag har aldrig läst någon förklaring på denna frågeställning. Kan ni förklara eller bevisa hur det ligger till??

Svar är väldigt tacksamt.

Mvh Simon B
/Simon B, Jönköping

Svar:
Simon! Vi har flera svar om tvillingparadoxen, se framför allt fråga 17238 och länkar från denna. Man behöver inte blanda in acceleration och allmänna relativitetsteorin, utan "paradoxen" kan förstås med begreppet egentid och den speciella relativitetsteorin.

Vilken riktning man färdas i har ingen betydelse, så dina båda resenärer som åkt åt olika håll är lika gamla när de kommer hem.
/Peter E

*

Blandat [19182]

Fråga:
Hej, jag undrar vad en dimension egentligen är. Jag vet att vi säger att vi lever i en tredimensionell värld och att Einstein sade att den fjärde dimensionen var tiden. I strängteorin säger man att vi har ännu fler dimensioner. Så vad är en dimension?
/Johanna L

Svar:
Dimension är inom geometrin ett uttryck för hur många värden som behövs för att bestämma ett läge inom en geometrisk storhet, se Dimension . Vi lever i tre rumsdimensioner t.ex. fram-tillbaka, höger-vänster och upp-ner.

I relativitetsteorin är rummet mycket nära kopplat till tiden, så det är naturligt att betrakta även tiden som en dimension, se Rumtid .

De extra dimensionerna i t.ex. strängteorierna kan vi inte föreställa oss, så vi får betrakta dem som matematiska konstruktioner.
/Peter E

*

Kraft-Rörelse [19394]

Fråga:
Om sambandet mellan Einsteins speciella respektive hans allmänna relativitetsteori, Einsteins gravitationsteori, får man ofta upplysningen att han konstruerade sin gravitationsteori genom att utgå från sin speciella teori som plattform för en utvidgning till den allmänna teorin. Detta stämmer dock inte om man får tro Einstein själv, som när hans gravitationsteori publicerats, beskrev hur han i sina ansträngningar att formulera den, först försökte använda den speciella teorin som plattform, men efter flera misslyckaden gav upp dessa ansträngningar att utvidga den speciella teorin. Einstein berättar att han nådde sitt mål först genom att släppa den speciella teorin och i stället angripa problemet förutsättningslöst. Upphovsmannen själv borde väl om någon ha vetat vad han talade om? Är det då en korrekt slutsats, att man inte måste vara bevandrad i den speciella teorin för att tillgodogöra sig både grundläggande och avancerade kunskaper helt inom ramen för den allmänna teorin och att när man väl nått den kunskapsnivån, så får man odiskutabelt den speciella teorin på köpet eftersom den utfaller som ett specialfall av den allmänna? Hur är det egentligen med den här saken?
/Carl Johan S, Lund

Svar:
Jag kan inte se varför den speciella relativitetsteorin (SR) och allmänna relativitetsteorin (AR) skulle utgöra en superteori. Man kan däremot hävda att namnet allmänna relativitetsteorin är olyckligt. Det vore bättre att kalla den Einsteins gravitationsteori. Som han säger började Einstein i princip från början när han tog fram den allmänna teorin.

Det är mycket stora skillnader mellan SR och AR. SR är matematisk mycket enkel (algebra). AR är matematiskt mycket svår (tensorkalkyl). Teorierna behandlar vitt skilda objekt: SR elementarpartiklar/atomer och AR astronomiska objekt och universum. Utgångspunkten för SR är relativitet och ljushastighetens konstans, för AR ekvivalensen mellan gravitation och acceleration.

Se vidare Theory_of_relativity#Two-theory_view och följande stycken.
/Peter E

Nyckelord: relativitetsteorin, allmänna [33]; relativitetsteorin, speciella [45];

1 http://www.svd.se/infallet-som-krokte-tiden-och-rummet/om/kultur:under-strecket

*

Kraft-Rörelse [19399]

Fråga:
Fenomenet att elektromagnetiska vågrörelser tar längre tid att gå genom ett gravitationsfält, än de skulle ta i frånvaro av detta fält, kallas ju gravitationell tidsfördröjning eller Shapirofördröjning. Einsteins gravitationsteori (AR) har förutsagt detta fenomen och det har gjorts många observationer som empiriskt bekräftat denna förutsägelse.

Enligt den speciella relativitetsteorin (SR) är ljushastigheten c invariant. Många som under sina högskolestudier endast studerat SR har svårigheter att acceptera att i AR är ljushastigheten icke invariant utan i regel antingen mindre eller större än c.

En annan svårighet som de möter är när de liksom Newton vill förklara att fotoner (Newron talade om korpuskler) på samma sätt som massa accelereras i att gravitationsfälts riktning då de passerar genom ett sådant fält från högre till lägre eller från lägre till högre gravitationspotential.

Att förklara att fotonerna, till följd av den energi som härleds ur deras frekvens, och till följd av ekvivalensen mellan massa och energi, är ”massekvivalenta”, är väl i grunden ett försök bortse från den förklaring som AR erbjuder på avböjningen, för att istället söka erkänna Newtons beräkning av densamma?

Enligt den allmänt accepterade förklaring av avböjningen som ges av AR, åstadkoms väl denna av att gravitation orsakar ett långsammare tidsförlopp vilket i sin tur påverkar ljushastigheten?

Är det över huvud taget relevant att söka analogiskt jämföra effekten av det långsammare tidsförloppet med effekten av det brytnings- eller refraktionsindex som är bestämmande för utbredningen av elektromagnetiska vågrörelser i ett medium?

Är alltså den hastighetsändring hos ett elektromagnetiskt vågtåg, som bl.a. kallas Shapirofördröjningen, en egenskap hos ett gravitationsfält som orsakar vågtågets deviation och som kan liknas vid ett ”brytningsindex” hos fältet? Med det undantaget att detta ”brytningsindex” är detsamma oavsett vilken våglängd som vågtåget har?

Den sista frågan ställer jag eftersom den avböjning, som ett elektromagnetiskt vågtåg ges vid sin passage förbi en tung kropp, är oberoende av våglängden hos vågtåget. Detta till skillnad från den uppdelning i olika färger som åstadkoms av ett prisma, då ljus av våglängder sammansatta i ett kontinuerligt spektrum får passera prismat.
/Carl Johan S

Svar:
Du har missuppfattat Shapiro-fördröjningen. Fördröjningen orsakas inte av att ljuset går långsammare utan av att vägsträckan i det krökta rummet är längre, se fråga 17427 .

Rödförskjutningen för små gravitationsfält kan härledas med klassisk fysik och bevarande av energin, se fråga 16989 .

Eftersom avlänkningen beror på rummets krökning påverkas alla fotoner lika oberoende av energi, se fråga 16021
/Peter E

*

Kraft-Rörelse [19450]

Fråga:
Jag tittade på "Into the universe with Stephen hawking", avsnittet med time travelling, men förstår inte riktigt. Hur och varför går tiden långsammare nära ett svart hål? Går den verkligen långsammare, borde det inte bara se ut som att den gör det därför att ljuset tar längre tid på sig att lämna det svarta hålet? Säg att vi har två tvillingar, den ena reser till ett svart hål vilket för denna person är 5 år. Tvillingen tittar på utifrån och för denna tar det tio år innan syskonet återvänder. Har då den ena tvillingen helt plötsligt blivit 5 år äldre, dvs har den andra tvillingen "rest i tiden"? Tid är ju något vi människor har "bestämt", och tvillingarna är ju födda samtidigt, så oavsett hur lång tid det känns som att det har gått för den enskilda personen så borde de båda ändå ha existerat lika länge? Sedan undrar jag också hur tiden går snabbare vid våra satelliter än nere på jorden, som det påstås i avsnittet. Min första tanke var att klockorna helt enkelt är konstruerade på jorden och därför kanske de annorlunda fysiska villkoren i rymden gör att klockorna går snabbare? För annars förstår jag inte konceptet, det är jättesvårt att greppa.
/Josefine M

Svar:
Josefine! Ja, det är svårt att förstå allmänna relativitetsteorin och svarta hål. Hawkins TV-program och böcker är inte alltid så lätta att förstå. Man får trösta sig med att det finns mycket omfattande experimentellt stöd för Einsteins teorier, se länkar nedan.

Ja, tiden går långsammare ju högre gravitationsfältet är, se fråga 14685 och 16989 . Se även länk 1 och General_theory_of_relativity#Gravitational_time_dilation_and_frequency_shift och Hafele–Keating_experiment .

Vad gäller tidsresor finns det inget observationellt stöd. Det är ett helt teoretiskt koncept, se fråga 19070 om maskhål. Tidsresor ger ju dessutom upphov till kausalitetsproblem (orsak/verkan). Tidsresa ger en bra intruduktion och Time_travel är mer omfattande.
/Peter E

Nyckelord: svart hål [51]; relativitetsteorin, allmänna [33];

1 http://www.upscale.utoronto.ca/GeneralInterest/Harrison/BlackHoles/BlackHoles.html

*

Ljud-Ljus-Vågor [19723]

Fråga:
Svänger E och B fälten i takt i elektromagnetisk strålning?
/Veckans fråga

Ursprunglig fråga:
Hej!

Jag läser just nu om elektromagnetiska svängningskretsar och hur radiovågor alstras, men det finns något jag inte riktigt förstår. Om man utgår från förklaringen med svängningskretsen där energi växlar mellan att vara elektrisk mellan kondensationsplattorna och magnetisk i spolen, kommer man fram till att det magnetiska fältet är minimalt när det elektriska fältet är maximalt och vice versa. Men när vi talar om elektromagnetiska vågor är inte fälten förskjutna relativt varandra med pi/2 rad, de är ju i fas!Jag har försökt googla runt och stötte på begreppen induktionsfält och strålningsfält, men ingen riktig ingående förklaring på vad dessa är och hur de gör att fälten hamnar i fas, så jag hade varit tacksam om jag hade fått en här!

Mvh, Rose
/Rose G, Katedralskolan, Växjö

Svar:
Engelska Wikipedia säger följande om elektromagnetisk strålning:

Electromagnetic radiation (EMR) is a form of radiant energy released by certain electromagnetic processes. Visible light is one type of electromagnetic radiation, other familiar forms are invisible electromagnetic radiations such as X-rays and radio waves.

Classically, EMR consists of electromagnetic waves, which are synchronized oscillations of electric and magnetic fields that propagate at the speed of light. The oscillations of the two fields are perpendicular to each other and perpendicular to the direction of energy and wave propagation, forming a transverse wave. Electromagnetic waves can be characterized by either the frequency or wavelength of their oscillations to form the electromagnetic spectrum. (Electromagnetic_radiation )

Maxwells ekvationer ger en vågekvation där amplituden på stort avstånd från källan (se nedanstående figur) avtar som 1/r. Detta är ditt strålningsfält. Fälten ser ut som i figuren i fråga 15035 . Man ser att E och B svänger i takt.

Nära källan är fälten mycket mer komplicerade eftersom vi har både laddningar och magnetism. Dessa (induktionsfältet) avtar emellertid hastigare än 1/r och försvinner på stort avstånd. Det är alltså bara strålningsfältet, som avtar som 1/r, som överlever på stort avstånd.

Det är alltså amplituden på fälten som avtar som 1/r. Energitransporten ges emellertid av Poyntings vektor (Poynting_vector#Plane_waves ):

P = konst*ExB = konst*E2/c,

och avtar, som sig bör, som 1/r2.

Se även Electromagnetic_radiation#Near_and_far_fields , Electromagnetic_radiation#Derivation_from_electromagnetic_theory och fråga 2867 .

Hoppas det blev lite klarare, Rose, men detta är inte helt lätt. Bra föreläsningar om Maxwells ekvationer och elektromagnetisk strålning av professor Shankar (Yale) finns under länk 1 och 2. Mot slutet av föreläsning 2 visar föreläsaren hur den magnetiska kraften uppkommer som en relativistisk effekt på laddningar som rör sig. Det var detta som fick Einstein att utveckla sin speciella relativitetsteori.



/Peter E

Nyckelord: elektromagnetisk strålning [21]; Maxwells ekvationer [3]; relativitetsteorin, speciella [45];

1 https://www.youtube.com/watch?v=yINtzw63Knc
2 https://www.youtube.com/watch?v=JJZkjMRcTD4

*

Kraft-Rörelse [19747]

Fråga:
Jag är förvisso varken lärare eller elev. Men jag har sett att svaret på en av frågorna verkar vara fel. Det gäller fråga 12834. I svaret står det:

"Hur kan vi förstå det faktum att den tröga massan och den tunga massan är lika? Jo, det är en naturlig följd av Einsteins allmänna relativitetsteori. Denna säger att gravitation är ekvivalent ("samma som") acceleration. Låt oss anta att vi har ett gravitationsfält som ger tyngdaccelerationen g (nedåt). Detta är då ekvivalent med en hiss som har accelerationen g (uppåt). Vad händer om du släpper en lätt kula och en tung kula mitt i hissen? I förhållande till hissen (som ju accelererar uppåt) rör sig kulorna hela tiden med samma hastighet (nämligen hastigheten noll eftersom de inte påverkas av någon kraft)."

I svaret står det att kulorna är stilla i förhållande till hissen som accelererar uppåt. Det kan väl inte stämma? Om hissen stod stilla så skulle kulorna falla till golvet. Om hissen accelererar uppåt så faller kulorna ännu snabbare till golvet. Medan om hissen "föll fritt" nedåt, så skulle kulorna däremot stå stilla i förhållande till hissen. Är det inte så som jag skrev, eller har jag uppfattat något på fel sätt?

Med vänlig hälsning

Tore
/Tore J, Se först i min fråga!, Stockholm

Svar:
Du refererar till fråga 12834 .

Nej, det står inte att kulorna är stilla i förhållande till hissen. De är stilla i förhållande till ett fixt eller icke accelererande system. Jag har försökt göra det klarare med en liten redigering. Det viktiga är att kulorna inte påverkas och att de således måste röra sig likadant i förhållande till hissen, dvs kulorna faller med samma acceleration enligt ekvivalensprincipen.
/Peter E

*

Universum-Solen-Planeterna [20117]

Fråga:
Gravitationsstrålning påvisad
/Veckans fråga

Ursprunglig fråga:
Vad tror du om rapporten om att man observerat gravitationsstrålning? Nobelpris eller misstag?
/Sven P

Svar:
Nobelpris i fysik 2017
Gravitationsvågor (både den populära och den avancerade artikeln är utomordentligt välskrivnen)
Rainer Weiss, Barry C. Barish, Kip S. Thorne
"för avgörande bidrag till LIGO-detektorn och observationen av gravitationsvågor"

Gravitationsvågor är inom fysiken krusningar i krökningen av rumtiden som propagerar som vågor som rör sig ut från källan. Fenomenet förutspåddes 1916 av Albert Einstein baserad på hans allmänna relativitetsteori, som säger att gravitationsvågor transporterar energi som gravitationsstrålning.

Data och analys ser mycket övertygande ut. En oberoende observation krävs nog för nobelpris, men med det är det ett kassaskåpssäkert nobelpris. Vilka av c:a 1000 författare som får det är en svårare fråga. Knappast Abbot som står först på bästa bokstavsordning.

Det finns flera skäl till att detta är en mycket viktig upptäckt:

  • Ännu ett bevis för att den allmänna relativitetsteorin är korrekt - exakt 100 år efter Einsteins publikation
  • Öppnar ett nytt fönster för observationer av universum
  • Det hittills mest direkta beviset för att svarta hål med en massa av storleksordningen tiotals solmassor existerar
  • Ger information om den senare utvecklingen hos massiva stjärnor
  • Gravitationsvågor har observerats i två detektorsystem: Hanford i Washington State (H1) och Livingstone i Louisiana (L1). Avståndet mellan dessa är c:a 3000 km, så den maximala skillnaden i ankomstid till detektorerna är 10 ms.

    Fördelen med flera detektorer långt ifrån varandra är dels att man kan eliminera lokala störningar och dels att man kan lokalisera källan. Med fler detektorsystem (som är på gång) kan man lokalisera källan mer exakt. Man kan då leta efter signaler i t.ex. röntgen, gamma eller det synliga området av det elektromagnetiska spektrum.

    Detektorerna består av en laser-interferometer med 4 km långa ben i 90 graders vinkel mot varandra. Idén är att gravitationsvågona påverkar benen olika vilket detekteras genom interferens vid startpunkten.

    Händelsen med det lite fantasilösa namnet GW150914 (gravitational wave 14/9 2015) tolkas som en sammansslagning mellan två svarta hål. Detta är nog den mest våldsamma händelse vi kan observera i universum. Energin i gravitationsvågorna är 3 solmassor dvs

    E = mc2 = 2*1030*(3*108)2 = 1.8*1047 J

    Detta var en av de händelser man hoppades kunna detektera, men man vet fortfarande inte hur vanliga dessa sammanslagningar mellan svarta hål är. Som hjälp i sökandet använder man simuleringar med den allmänna relativitetsteorin av svarta hål av olika storlek som slås ihop. Signalen man väntar sig visas näst längst ner i nedanstående bild. När hålen kommer nära varandra kommer de att sända ut mycket energi bland annat i form av gravitationsstrålning. De kommer då att spinna snabbare och snabbare runt varandra. Frekvensen ökar och amplituden ökar tills händelsehorisonterna överlappar och de båda svarta hålen blir ett. Hela processen tar c:a 0.2 sekunder.

    Övre bilden visar data från GW150914. Den röda kurvan (från H1) har förskjutits 7 ms och inverterats. Man ser att det är en imponerande överensstämmelse med kurvan från L1-detektorn. Överensstämmelsen med de teoretiska beräkningarna är också mycket god. Man har även bestämt massorna av de två svarta hålen till 36 och 29 solmassor. Det resulterande svarta hålets massa bestämdes till 62 solmassor. De resterande 3 solmassorna blev alltså gravitationsstrålningen som observerades.

    För att man skall vara säker på att de som analyserar data gör ett bra jobb lägger man in testdata i smyg:

    "In the past, a few senior members of the LIGO team have tested the group's ability to validate a potential discovery by secretly inserting ‘blind injections’ of fake gravitational waves into the data stream to test whether the research team can differentiate between real and fake signals. But the September detection happened before blind injections were being made, so it is thought to be a signal from a real astrophysical phenomenon in the Universe."

    Den officiella annonseringen av upptäckten finns i länk 2. Se fråga 19870 för mer om kolliderande svarta hål. Se även Gravitational_wave och Gravitational_wave_observation .

    Bilder från föredrag av Chris Van Den Broeck: The direct detection of gravitational waves: The first discovery, and what the future might bring
    http://indico.lucas.lu.se/getFile.py/access?resId=0&materialId=slides&confId=333

    Tillägg 16 okt. 2017

    "The merging of two neutron stars was detected by gravitational waves and then by telescopes in all parts of the electromagnetic spectrum."



    /Peter E

    Nyckelord: gravitationsvågor [19]; svart hål [51]; nyheter [11];

    1 http://www.nature.com/news/has-giant-ligo-experiment-seen-gravitational-waves-1.18449
    2 http://journals.aps.org/prl/abstract/10.1103/PhysRevLett.116.061102

    *

    Blandat [20136]

    Fråga:
    Kunskapsmål i fysik
    /Veckans fråga

    Ursprunglig fråga:
    Jag är lite nyfiken på hur ni experter inom fysik skulle tolka följande textrad hämtad ur LGR11, kunskapsmålen i fysik för åk 6:

    "Dessutom förklarar eleven och visar på mönster i himlakroppars rörelse i förhållande till varandra..."

    Det är ett kriterium för betyget A. Det som skiljer målen för A från kunskapsmålet på nivå C (som är lägre) är att då behöver eleven istället för att se mönster i rörelserna visa på samband kring himlakroppars rörelse i förhållande till varandra.

    Jag har undervisat eleverna i min åk 5 om hur bl.a. planeterna rör sig runt solen och hur månar rör sig kring planeter. Vi har pratat om att gravitationen håller planeter och månar i sina omloppsbanor och att gravitationen påverkar kroppar som är nära mer än kroppar som befinner sig långt ifrån. Vi har pratat om att det är anledningen till att planeterna närmast solen rör sig snabbare i sina omloppsbanor jämfört med planeterna längre bort. Generaliseringen som vi gjort är att himlakroppar med större massa och gravitation håller kroppar med mindre massa och gravitation i omloppsbana istället för tvärtom. Men vi har också resonerat om att de mindre kropparna också drar i de större och skapar en "wobblande" rörelse hos dem.

    Hur tolkar ni skillnaden mellan att se mönster och se samband kring himlakropparnas rörelser. Är det något som jag borde ta upp mer med mina elever för att komma ner på det djup som kunskapsmålet riktar in sig på?

    Med vänlig hälsning Petri Matalamaa
    /Petri M, Paulinska skolan, Strängnäs

    Svar:
    Hej Petri! Jag håller i stort med om kunskapsmålen och tycker det låter som att du uppfattat dem mycket bra.

    För det första är jag allergisk mot begreppet förklara när det gäller fysikens lagar. Förklara är för mig svar på frågan varför?. I djupare mening vet vi inte varför naturlagarna är som de är, se fråga 12126 .

    Fysik är en empirisk vetenskap som i grunden bygger på observationer och experiment, se fråga 14232 . Fysiken använder matematik som ett verktyg, men fysik är inte matematik. Ofta utgår man från ett antagande och detta antagande kan sedan få stöd genom direkta eller indirekta observationer. Einstein antog t.ex. att ljushasigheten i vakuum är konstant när han utvecklade sin speciella relativitetsteori, och denna har visat sig stämma mycket väl.

    Från Tycho Brahes mycket exakta mätningar av planeten Mars' rörelse (slutet av 1500-talet) kunde Johannes Kepler (i början av 1600-talet) få fram tre lagar för planeternas rörelser. Samtidigt använde Gallileo Gallilei det nyuppfunna teleskopet för att göra astronomiska observationer. Han studerade även, både teoretiskt och experimentellt, kroppars rörelse.

    Isaac Newton kunde senare (andra hälften av 1600-talet) "förklara" planeternas rörelser med hjälp av en lag, den universella gravitationslagen och nyutvecklad matematik (differentialkalkyl).

    Den ovanstående utgör det centrala i utvecklingen av den moderna vetenskapliga metod som används i naturvetenskapen, se fråga 14237 .

    En annan viktig aspekt på fysik är att fysikaliska lagar inte är huggna i sten, utan de kan modifieras allteftersom vi gör bättre observationer.

    Lagarna kan emellertid inte ändras hur fritt som helst, utan de måste alltid kunna reproducera alla befintliga mätresultat. Einsteins allmänna relativitetsteori beskriver gravitationen på ett utmärkt sätt, men den är mycket olik Newtons gravitationsteori. Det betyder inte att Newton hade fel, bara att det fanns begränsningar i giltigheten. Man använder t.ex. forfarande Newtons teori för att beräkna banor för rymdsonder.

    I fysik använder man sig ofta av förenklade modeller som beskriver ett fysikaliskt fenomen med begränsade förutsättningar, se fråga 18296 . I kärnfysik betraktar man t.ex. ibland atomkärnan som en vätskedroppe och ibland som nukleoner som rör sig fritt i en potential (skalmodell). Modellerna är egentligen helt inkompatibla, men de är ändå av värde eftersom de båda "förklarar" olika egenskaper hos atomkärnor.

    Och nu äntligen till din fråga:

    Begreppet "förklarar" i texten ovan får man tolka så att man kan beskriva ett fenomen på en djupare nivå. Låt oss ta solens rörelse. Den dagliga rörelsen från öster till väster "förklaras" av att jorden roterar kring sin axel.

    Solen rör sig även lite från väster till öster i förhållande till de avlägsna stjärnorna. Detta "förklaras" av att jorden rör sig ett varv runt solen på ett år. För att få ett A måste vi även kunna beskriva att den årliga rörelsen beskrivs av Newtons gravitationslag.

    Den "wobblande" rörelsen du talar om uppkommer eftersom två kroppar rör sig i elliptiska banor kring den gemensamma tyngdpunkten. Om mass-skillnaden mellan kropparna är stor ligger den gemensamma tyngdpunkten nära den tyngre kroppens centrum. Detta betyder att den tyngre kroppen rör sig mycket lite.

    Ett annat exempel är solsystemets uppbyggnad. Vi kan observera att planeterna alla rör sig åt samma håll och i ett plan. Detta kan "förklaras" genom modellen att solsystemet bildats genom att ett gasmoln dras samman av gravitationen. Bevarande av rörelsemängdsmomentet (se fråga 12527 ) ger då upphov till en roterande skiva av gas och stoft. Denna bildar sedan planeterna i ett plan och rörelse åt samma håll.

    I fråga 16776 beskrivs solsystemets rörelse i vintergatan och universum.
    /Peter E

    Nyckelord: fysik, förståelse av [17]; vetenskaplig metod [18]; fysik [10]; fysikalisk modell [12]; kursplan [3];

    1 http://pimlab.learnify.se/learnifyer/ObjectResources/f9c9e402-4879-4502-8f4c-ee6fc78fb396/Kunskapskrav.pdf

    *

    Kraft-Rörelse [20167]

    Fråga:
    Hej! Jag håller på med en inlämningsuppgift om den speciella relativitetsteorin och har fastnat på två uppgifter.

    1: Varför kan inget vara snabbare än ljuset?

    2:Hur påverkas rörelseenergin och rörelsemängden av den speciella relativitetsteorin? På denna har jag skrivit följande " I vanliga fall så gäller formeln E_k=(mv^2)/2 för rörelseenergin, men precis som med längder och tider spelar hastigheten in på föremål som rör sig. Därför måste vi använda oss av gammafaktorn. Enligt den speciella relativitet teorin är massa och energi samma sak, och beskrivs med E=mc^2  E=γmc^2=(mc^2)/√(1-v^2/c^2 )=Den totala energin Om hastigheten är 0 får vi E_0=(mc^2)/√(1-0^2/c^2 )=(mc^2)/1=mc^2=Viloenergin Om vi subtraherar Den totala energin med viloenergin så får vi rörelseenergin. E_k=E-E_0  E_k=(mc^2)/√(1-v^2/c^2 )-mc^2" Men det känns som jag är helt fel ute

    3 När ska man lyssna på Newton och när ska man lyssna Einstein? Jag har gjort en härledning på rörelseenergin för att få fram att Newton går bra när hastigheterna är låga. Stämmer det?


    /Axel P, It-Gymnasiet, Skövde

    Svar:
    1 Se fråga 18067 .

    2 Uttrycken ser OK ut men det hade varit tydligare om du kallat vilomassan m0. Se fråga 14250 .

    3 Ja, om v/c är mindre än 0.1 ger det klassiska uttrycket bra resultat. De relativistiska uttrycken är alltid korrekta, men man får lite mer komplicerade räkningar.
    /Peter E

    *

    Kraft-Rörelse [20173]

    Fråga:
    En fråga om relativitetsteorin,jag menar att det är bluff det här med att tiden skulle gå långsammare ju snabbare man färdas. Om vi har två klockor,som mekaniskt går exakt lika,typ atomur. Och sätter den ena i nån farkost med ljusets hastighet i ett år,och sen jämför den med den som vart kvar på jorden och det skulle vara skillnad på dom? Eller för den delen,vi människor lever ju i ett antal hjärtslag,om vi har två tvillingar som fysiskt leve lika länge,hur skulle den enas hjärta kunna slå så många fler slag än den andres? Kom ihåg att hjärtslag pumpas med en rytm som vi beskriver som ungefär 60 slag i minuten. Jag menar att allt är bluff,kan ni förklara så att jag kan tro på Einstein? Mvh Bror
    /Anna W, Gymnasiet, Hudiksvall

    Svar:
    Nej, det är inte bluff. Tidsdilatationen är en mycket väl etablerad effekt. Man har utfört det experiment du beskriver, men naturligtvis med lägre hastighet, se fråga 18882 och 14685 .
    /Peter E

    *

    Kraft-Rörelse [20414]

    Fråga:
    Mörk materia
    /Veckans fråga

    Ursprunglig fråga:
    Hej! Min fråga angår mörk materia.

    Om jag förstått det rätt så kan man visualisera "rummet" som kuber och att materia böjer kuberna mot sitt centrum och ger upphov till gravitation. Skulle det vara möjligt att någon händelse eller liknande gjorde att kuberna blev konstant böjda och det är då det som vi ser som mörkmateria?
    /Fredrik A, Ingen, Kållered

    Svar:
    Det är korrekt att massa kröker rymden - det är ett grundläggande resultat av den allmänna relativitetsteorin. Det finns teoretiker som har spekulerat att mörk materia inte finns utan bara är en spontan krökning hos rymden, se t.ex. Dark_matter#Alternative_theories . Problemet är att man måste ge sig på att modifiera Einsteins ekvationer, och det för med sig nya problem.

    I måndags (21/11 2016) sändes ett inslag i Vetenskapens värld om mörk materia, se länk 1. Där förutsägs att man rett ut problemet med den mörka materien inom fem år. Vi kan alltid hoppas.

    Se fråga 13626 och 9324 för mer om relativitetsteorin och fråga 20164 och 12396 för mörk materia. Bilden nedan är från länk 2 med bildtexten:

    In this image, dark matter (blue) has become separated from luminous matter (red) in the bullet cluster. Image courtesy of Chandra X-ray Observatory.



    /Peter E

    Nyckelord: relativitetsteorin, allmänna [33]; mörk materia [17];

    1 http://www.svt.se/nyheter/vetenskap/forskare-vi-ar-nara-att-hitta-mork-materia
    2 http://home.slac.stanford.edu/pressreleases/2006/20060821.htm

    *

    Kraft-Rörelse [20638]

    Fråga:
    Hej,

    Einstein menar på att planeter med stor massa böjer rumtiden i rymden. Vi brukar visa det genom att lägga en tung boll på en utsträck duk där duken böjs. Den tunga bollen dras ner tack vare jordens gravitation. Då tänker jag att ute i rymden när planeter böjer rumtiden så måste det även där finnas någon annan typ av gravitation så att planeterna har den möjligheten att böja rumtiden precis så som bollen kan böja duken pga jordens gravitation. Den tunga bollen skulle alltså inte böja duken om inte jordens gravitation fanns. Frågan är då, hur kan planeter med stora massor böja rumtiden utan en annan gravitation? Eller finns den?

    Hälsningar

    Marvin Sanchez
    /Marvin S, Högskolan i Gävle, Byggnadsingenjör

    Svar:
    Gravitation (av latin gravis = tung) även känd som tyngdkraft är en av universums fyra fundamentala krafter. Det är den attraherande kraft som massor utsätter varandra för, och ger upphov till det som vi kallar massans tyngd.

    Med den allmänna relativitetsteorin beskrevs gravitationen som en krökning av rummet (och tiden), och vad vi ser som en kraft som får massor att accelerera mot varandra är då en direkt konsekvens av att de färdas i ”räta linjer” i denna böjda rumtid. I en populär modell tänker man sig en uppspänd gummiduk på vilken massorna ligger och orsakar att duken i närheten sjunker ned en bit. (Gravitation )

    Bilden med ett gummimembran som deformeras (se nedan) genom att man placerar en boll på det är inte bra. Kraften som deformerar membranet är ju den klassiska newtonska gravitationskraften. Det är bättre att säga att massa deformerar rum-tiden och detta orsakar gravitationen. Det finns alltså i Einsteins teori ingen kraft utan en uppsättning ekvationer som ger krökningen som funktion av massan som sägs i serien nedan från länk 1.

    Ekvationen längst ner (från Introduction_to_general_relativity ) ser oskydigt enkel ut, men det kräver tyvärr avancerade kunskaper i matematik.

    I fråga 17427 beskrivs hur man kan mäta rymdens krökning nära en massa.

    Eftersom Einsteins gravitationsteori är lite svår att hantera matematiskt använder man för många tillämpningar fortfarande Newtons gravitationslag, se fråga 12834 .



    /Peter E

    Nyckelord: relativitetsteorin, allmänna [33]; Newtons gravitationslag [12]; gravitation [7];

    1 https://physics.stackexchange.com/questions/220873/is-the-popular-explanation-given-for-gravity-in-general-relativity-misleading
    2 https://www.space.com/17661-theory-general-relativity.html

    *

    Kraft-Rörelse [20773]

    Fråga:
    Hej jag undrar hur krökningen fungerar. Om det läggs ett bowlingklot i en duk och det bildas en krökning i den. Varför åker pingisbollen runt där tills den stannar pågrund av friktionen. Förstår att den följer en bana men varför åker den inte direkt ned?
    /Madeleine H, färentuna

    Svar:
    Madeleine!

    Jag har aldrig varit förtjust i modellen för gravitation med en elastisk duk och ett tungt klot, se animeringen nedan från länk 1. En svaghet är att man använder gravitation för att demonstrera gravitation - det är ju gravitationen på den stora kulan som skapar krökningen som representerar gravitationen. Se vidare diskussion i fråga 20638 .

    Det grundläggande problemet är emellertid att man inte kan separera rum och tid, se länk 2. Detta gällde redan i den speciella relativitetsteorin. Till skillnad från Newtons mekanik, där man har en universell tid, måste man i Einsteins teori använda en fyrdimensionell rum-tid. Det är svårt att föreställa sig en fyrdimensionell rum-tid. Det blir inte bättre av att massa i den allmänna relativitetsteorin orsakar en krökning i rum-tiden.

    Anledningen till att den lilla kulan inte åker direkt mot centrum är att rörelsemängdsmomentet L=mvr måste bevaras, se fråga 12527 . Om kulan har en hastighetskomponent vinkelrätt mot riktningen mot centrum kan den inte (utan friktion) gå genom centrum.

    Om man skjuter den lilla kulan rakt mot centrum är L=mv*0=0, och kulan kan gå genom centrum.

    Se även fråga 17427 .



    /Peter E

    Nyckelord: gravitation [7]; relativitetsteorin, allmänna [33];

    1 https://spaceplace.nasa.gov/what-is-gravity/en/
    2 http://curious.astro.cornell.edu/physics/140-physics/the-theory-of-relativity/general-relativity/1023-if-gravity-is-a-curvature-of-space-rather-than-a-force-why-do-a-ball-and-bullet-follow-different-paths-intermediate

    *

    Kraft-Rörelse [20847]

    Fråga:
    Angående Einsteins relativitetsteori… Nu håller jag på och läser en bok av Jan Slowak som ifrågasätter den speciella relativitetsteorin. Jag hade ju, på 60-talet, en genial fysiklärare som engagerade sig i Einsteins teorier och han menade att de var felräknade. När E:s teorier inte stämde så lade E in konstanter så att de ändå stämde men ändå var fel. E drog sig inte ens för att göra om Pythagoras sats… E satte ett minustecken framför den kortaste katetens kvadrat… Den här läraren hävdade att om man räknar rätt så utvidgar sig inte universum eftersom rödförskjutningen och blåförskjutningen är lika stor! Precis dessa saker kan man hitta i boken ”Einsteins speciella relativitetsteori = matematiskt och fysikaliskt nonsens!”

    Jag minns att när vi frågade vår lärare vad hans upptäckter medförde så sa han att om mänskligheten tror att universum utvidgas trots att det inte sker så spelar det absolut ingen roll. All astronomi är hjärnföroreningar! Om vetenskapsmän uppfattar Einstein som en gud så ser de inte de uppenbara fel som finns i hans teorier. Eller också är det så att de inte vill se felen för då förlorar de alla karriärmöjligheter…

    Jag har försökt att hitta kommentarer till Jan Slowaks forskning utan att lyckas. Det som känns konstigt är att felräkningarna som min lärare påpekade på 60-talet finns beskrivna i Jan Slowaks bok. Kan det vara så att fortfarande ingen forskare vågar se Einsteins felräkningar? Tyvärr så omkom min geniale lärare i en trafikolycka något år senare. /Harald
    /Harald G, Östersund

    Svar:
    Einsteins speciella relativitetsteori (SR) är - till skillnad från den allmänna relativitetsteorin (AR) - matematiskt närmast trivial. I fråga 20459 visas t.ex. en härledning av tidsdilatationen som bara kräver elementär geometri/algebra.

    Den speciella relativitetsteorin bygger på två mycket rimliga antaganden, se fråga 16263 . Det finns dessutom ett otal experiment som stöder teorin, se t.ex. fråga 2697 och 14685 .

    Det råder stor enighet i vetenskapssamhället att SR och även AR ger en mycket bra beskrivning av det vi kan observera. De utgör inte den slutgiltiga teorin eftersom integreringen av AR med kvantmekaniken saknas.

    Det finns flera exempel på författare som kritiserat och gjort om delar av den moderna fysiken. Det mest omfattande exemplet är Randell Mills som skrivit en bok på över 1000 sidor med värdelöst nonsens, se fråga 14237 och Brilliant_Light_Power#Criticism . Jan Slowak är på en helt annan lägre nivå, se länk 1. Du har rätt i att det är svårt att hitta någon som tagit den minsta notis om JSs teorier som handlar dels om SR och dels om Big Bang. Dock har en professionell fysiker (anonym?) tagit sig tid att vänligt kritisera JSs teorier, se länk 2.

    Sannolikheten att en person i dag skulle hitta grundläggande fel i en etablerad teori som SR är om inte noll så åtminstone mycket nära noll. Einstein själv framställs ibland som en isolerad kuf med revolutionerande idéer. Detta är långt ifrån sanningen. Han hade mycket bra kontroll på den aktuella vetenskapliga utvecklingen de första åren på 1900-talet. SR var en frukt av flera fysikers arbeten, men det var Einstein som "knöt ihop säcken" och skapade en konsistent teori.
    /Peter E

    Nyckelord: relativitetsteorin, speciella [45]; pseudovetenskap [11];

    1 https://www.bokus.com/cgi-bin/product_search.cgi?authors=Jan%20Slowak
    2 https://www.smakprov.se/smakprov/visa/9789174637533/partner/smakprov

    *

    Universum-Solen-Planeterna [21202]

    Fråga:
    Existensen av svarta hål.
    /Veckans fråga

    Ursprunglig fråga:
    1. Hur mycket bevis krävs för att man säkert ska kunna fastställa att man har hittat ett svart hål?

    2.Vilken detektionsmetod ger mest övertygande bevis för svarta håls existens?

    3.Och vilka för-och nackdelar med de olika metoderna finns det?
    /Mina M, Europaskolan Strängnäs, Eskilstuna

    Svar:
    1 Det beror på hur "godtrogen" man är. Till och med Steven Hawking var länge tveksam, men gav upp motståndet i slutet på 1980-talet, se länk 1 och Hawkings bok A Brief History of Time (1988).

    Nu (2020) finns så mycket bevis, se a-e nedan, att tvivlare närmast kan jämföras med klimatförnekare.

    2 Att följa stärnor i galaxcentum och beräkna massan med Keplers tredje lag.

    3 Det får du fundera på själv med hjälp av Black_hole#Observational_evidence .

    Här är en lista på några ganska övertygande bevis att svarta hål (se fråga ) existerar och att de har observeras.

    a Allmänna relativitetsteorin

    Existensen av svarta hål är en konsekvens av Einsteins Allmänna relativitetsteori (1916). Denna teori har testats på många sätt och med stor precision. Teorin har i alla fall visat sig stämma med observationer.

    Se fråga 12745 och General_relativity#Black_holes_and_other_compact_objects .

    b Aktiva galaxer och andra objekt som sänder ut röntgenstrålning

    Dessa objekt har det gemensamt att de sänder ut enorma mängder röntgenstrålning när materia från en ackretionsskiva faller in i ett svart hål. Små objekt är ofta dubbelstjärnesystem där komponenterna har en massa av tiotals solmassor.

    Stora objekt (massa milliontals solmassor) befinner sig oftast i centum av en galax, och sänder ut enorma mängder röntgenstrålning. Den utsända effekten är så stor att den enda rimliga förklaringen är att vi har att göra med ett svart hål som slukar materia. Upp till 50% av massan kan förvandlas till strålning i ett sådant objekt (se fråga 14367 ).

    Se fråga 13916 , Active_galactic_nucleus och Cygnus_X-1 .

    c Observationer av stjärnor som kretsar kring ett objekt i centrum av en galax

    Genom att följa stjärnor i centrum av vintergatan kan man räkna ut massan på objektet som finns i centrum. Resultatet blir en så stor massa i ett litet område att den rimligaste förklaringen är att vi har att göra med ett svart hål.

    Se fråga 6228 och Sagittarius_A*#Orbiting_stars .

    d Observation av kolliderande svarta hål med gravitationsstrålning

    Man har sedan 1950-talet med allt större apparater försökt att detektera gravitationsstrålning. Det man i första hand detekterar är våldsamma rörelser hos stora massor. En av de mest våldsamma händelser man kan tänka sig är att två svarta hål kolliderar och slås samman till ett. Detta lyckades man observera för några år sedan.

    Se fråga 20117 och gravitational_wave_observation .

    e Direkt avbildning av ett svart hål

    Ja, här måste vi säga att vi har tillräckliga bevis för brottet: brottslingen är fångad på bild, se nedan. Nu kan man tycka att det är konstigt att man kan fotografera ett svart hål. Vad man ser på bilden är inte det svarta hålet (händelsehorisonten, se Event_horizon ) utan skuggan av hålet. Strålningen vi ser (som är radiovågor, se nedan) kommer från materia som faller ner i hålet - delvis faktiskt från bakom hålet.

    Ett svart hål är mycket litet så man behöver ett teleskop med hög vinkelupplösning. Detta åstadkommer man genom att kombinera data från flera olika radioteleskop spridda över jorden. På så sätt kan man åstadkomma en vinkelupplösning som motsvarar ett teleskop med en radie motsvarande jordens radie.

    Se länk 2 och Black_hole#Observational_evidence .



    /Peter E

    Nyckelord: svart hål [51]; relativitetsteorin, allmänna [33]; händelsehorisont [4];

    1 https://en.wikipedia.org/wiki/Thorne%E2%80%93Hawking%E2%80%93Preskill_bet#Earlier_Thorne–Hawking_bet
    2 https://eventhorizontelescope.org/press-release-april-10-2019-astronomers-capture-first-image-black-hole

    *

    Ljud-Ljus-Vågor [21315]

    Fråga:
    Hej! Gravitationsstrålning, finns någon formel för dess strålning? (Elektromagnetisk strålning ha ju E = hf, men hur ser det ut för gravitaitonsstrålningen?)
    /Thomas Å, Knivsta

    Svar:
    Nej, det finns ingen enkel formel.

    Gravitationsvågor är en deformation i rum-tiden (se fråga 20117 ) som bildas av accelererade massor, se animeringen nedan.

    Beskrivningen av gravitationsvågor kräver hela tensor-apparaten i den allmänna relativitetsteorin.

    En tensor är ett matematiskt objekt som är en generalisering av begreppen skalär, vektor och linjär operator. Tensorer är betydelsefulla inom differentialgeometri, fysik och teknik. ... Einsteins allmänna relativitetsteori, utvecklad under 1910-talet, formuleras med hjälp av tensornotation, (Tensor ) Gravitationsvåg

    Se även fråga 20638 , länk 1 och länk 2.

    Här är en animering av gravitationsvågor från två svarta hål som slås samman:

    An artist’s animation of gravitational waves created by the merger of two black holes. Credit: LIGO/T. Pyle
    /Peter E

    Nyckelord: gravitationsvågor [19]; relativitetsteorin, allmänna [33]; svart hål [51];

    1 https://spaceplace.nasa.gov/gravitational-waves/en/
    2 https://www.sciencemag.org/news/2016/02/gravitational-waves-einstein-s-ripples-spacetime-spotted-first-time

    *

    Sök efter    

    Skriv de ord du vill söka på i sökfältet ovan och klicka på sökknappen. Uteslut ord genom att sätta - (minus) före ordet. Ordgrupper definieras med hjälp av "...". Sökningar är oberoende av stora och små bokstäver.

    Exempel:

    helium "kalle anka"
    Sök på 'helium' och ordgruppen 'kalle anka'
    orgelpipa
    Sök på 'orgelpipa'
    orgel -gitarr
    Sök på 'orgel' men inte 'gitarr'

     


    sök | söktips | Veckans fråga | alla 'Veckans fråga' | ämnen | dokumentation | ställ en fråga
    till diskussionsfora

     

    Creative Commons License

    Denna sida från NRCF är licensierad under Creative Commons:
    Erkännande-Ickekommersiell-Inga bearbetningar
    .